Вы находитесь на странице: 1из 81

GRE Verbal Study Guide

Table of Contents
Chapter 1 Reading Comprehension ...............................................................................3
Section 1: One Principle .............................................................................................. 4
Section 2: Two Styles .................................................................................................. 7
1. Presentation....................................................................................................... 7
2. Argumentation .................................................................................................. 8
3. Organizational Structure ................................................................................. 10
Section 3: Three Subjects........................................................................................... 13
1. Natural Science ............................................................................................... 13
2. Social Science ................................................................................................. 15
3. Business Subject ............................................................................................. 17
Section 4: Four-step Process of Reading ................................................................... 19
1. Analyze the first paragraph............................................................................. 19
2. Skim the passage and get the author's main point........................................... 20
3. Diagram the organization of the passage ........................................................ 22
4. Tackle the questions and correspondently refer to the passage. ..................... 25
Section 5: Five Types of Questions ........................................................................... 28
1. Main Idea Question......................................................................................... 29
2. Recall Question............................................................................................... 34
3. Inference Questions ........................................................................................ 36
4. Critical Reasoning Question ........................................................................... 38
5. Difficult-to-locate Question ............................................................................ 40
Section 6: Six test points............................................................................................ 42
1. Comparison ..................................................................................................... 42
2. Example & Listing.......................................................................................... 44
3. People, Date & Place ...................................................................................... 45
4. Words of Attitude and Transition ................................................................... 47
5. Counter-evidence Indicators ........................................................................... 48
6. Special Punctuation......................................................................................... 50
Review ....................................................................................................................... 51
Chapter 2 Sentence Completion...................................................................................52
Section 1: The four types of relationship ................................................................... 53
1

A. Contrast .......................................................................................................... 53
B. Cause and Effect............................................................................................. 55
C. Explanation..................................................................................................... 57
D. Similarity........................................................................................................ 59
Section 2: Five-Step Procedure to Complete a Sentence........................................... 60
Chapter 3 Analogy .......................................................................................................61
Introduction................................................................................................................ 61
How to define the relationship ................................................................................... 62
Eighteen common types of relationship..................................................................... 63
Chapter 4 Antonyms ....................................................................................................76
Vocabulary Roots....................................................................................................... 77

Prep for GRE in Computer Adaptive-Test Format


If you have any question or suggestion, please email us at help@grecat.com.
Copyright 2004 The GRECAT.com. All Rights Reserved. Not for distribution.

Chapter 1 Reading Comprehension


Reading Comprehension on the test day
On the test day, you will expect to see two or three reading passages, each followed with three or
four questions. The passages presented depend on how well you are performing on the test.
However, the questions presented for the same passage do not depend on your performance. In
other word, after you are assigned a reading passage, the next question presented for the same
passage will not base on your performance on the previous question.
Why Reading Comprehension is a nightmare to most students?
Most people find the reading compression difficult to prepare because the subject matter is
unfamiliar and could be anything. In order to make sure that nobody can take advantages on a
particular subject, the test-maker takes every effort to diversify the subjects of the three or four
passages on your test day. As a result, obscure subject matter is chosen so that you will be tested,
not on your knowledge of a particular subject, but the test-taking skills.
In addition, the reading passage is not created like the one we see on magazine, newspaper, or
textbook. Rather, it uses a highly compressed style. Subjects of the passages are generally
excerpted from academic articles that were published tens of years ago. Usually the chosen
article is heavily edited until it is cut down to about 300 words, about one-third its original length.
Though it is difficult to read, the reading techniques introduced in the following passage will help
you pick up the right answer even without understanding the reading passage.
Chapter Preview
In order to make it easier for you to prepare for GRE, we have developed an interesting course for
Reading Comprehension. You will find this chapter all in number, as the section number suggests.
We hope this would help you learn the test prep strategies.
Section 1: One Principle
Section 2: Two Writing Styles
Section 3: Three Subjects
Section 4: Four-step Procedure for Attacking a Passage
Section 5: Five Types of Question
Section 6: Six Test Points

Section 1: One Principle


Directions: The questions in this group are based on the content of a passage. After reading the
passage, choose the best answer to each question. Answer all questions following the
passage on the basis of what is stated or implied in the passage
On the test day, you will see the above direction on computer screen. Most students disregard
this instruction since it appears in every test. However, it introduces a basic principle you should
follow in answering a reading comprehension question. When answering questions, you must
refer each of them to some place in the passage. Don't rely on memory, since too many traps are
used with these questions.
Also, don't base on your daily life experiences or college knowledge. Remember, GRE doesn't
test any specific knowledge on business or other functions. Image if a question is based on some
specific knowledge, then those with broad knowledge can take advantages. This definitely
violates the rule of GRE. The test-maker often fools you by creating stuff choices that contain
reasonable statement based on basic knowledge or your life experience, not on the passage. If
you find an answer choice contains the widely known reasoning or statement on the test day,
eliminate those choices with hesitation.
Let's look at a sample passage that discusses why the Indian software vendors perform better
than their counterparts in China.
Indian firms have achieved the highest levels of efficiency in the world software outsourcing
industry. Some researchers have assumed that Indian firms use the same programming
languages and techniques as Chinese firms but have benefited from their familiarity with
English, the language used to write software code. However, if this were true, then one
would expect software vendors in Hong Kong, where most people speak English, to perform
not worse than do Indian vendors. However, this is obviously not the case.
Other researchers link high Indian productivity to higher levels of human resource investment
per engineer. But a historical perspective leads to a different conclusion. When the two top
Indian vendors matched and then doubled Chinese productivity levels in the mid-eighties,
human resource investment per employee was comparable to that of Chinese vendors.
Furthermore, by the late eighties, the amount of fixed assets required to develop one
software package was roughly equivalent in India and in the China. Since human resource
investment was not higher in India, it had to be other factors that led to higher productivity.
A more fruitful explanation may lie with Indian strategic approach in outsourcing. Indian
software vendors did not simply seek outsourced contract more effectively: they made
aggressive strategic in outsourcing. For instance, most software firms of India were initially
set up to outsource the contract in western countries, such as United States. By contrary,

most Chinese firms seem to position their business in China, a promising yet
under-developed market. However, rampant piracy in China took almost 90 percents of
potential market, making it impossible for most Chinese firms to obtain sufficient
compensation for the investment on development and research, let alone thrive in
competitive environment.
Now, let's look at a sample question:
Which of the following statements concerning the productivity levels of engineers can be
inferred from the passage?
(A) Prior to the 1980s, the productivity levels of the top Indian software firms were exceeded
by those of Chinese software firms.
(B) The official language of a country has a large effect on the productivity levels of its software
developers.
(C) During the late 1980s and early 1990s, productivity levels were comparable in China and
India.
(D) The greater the number of engineers that a software firm has, the higher a firms
productivity level.
(E) The amount of human resource investment made by software developers in their firms
determines the level of productivity.
If you do not refer to the original passage, you may pick up B. For test-takers who have some
backgrounds in computer, it is obvious that being familiar with English will gain some advantage
in writing program code. However, the correct answer is C.
In conclusion, the directions can run out of your eyes on the test day, but should be rooted deeply
in your heart at the beginning of your test preparation.

Trap: Some choices just repeat the same words or phrases that you read in the passage. Keep
alarm to these choices since in most cases, they are incorrect.
Here is an example:
The fact that reducing price can generate a competitive advantage for a company does not
mean that every reduction in price will create such an advantage. Price reduction, like
improvement in service, must be balanced against other types of efforts on the basis of direct,
tangible benefits such as increased revenues. If a company is already effectively on a par
with its competitors because it provides product at an acceptable price and keeps customers

from leaving at an unacceptable rate, then reduction in price may not be effective, since price
is not necessarily the deciding factor for any customer in any situation.
This truth was not apparent to managers of one operating system software vendor, which
failed to improve its competitive position despite its attempt to reduce price. The software
managers did not recognize the level of customer inertia that arises from the inconvenience
of switching operating system. Nor did they analyze their reduction in price to determine
whether it would attract new customers by producing a new standard of price that would
excite customers or by proving difficult for competitors to copy.
Sample question
According to the passage, reduction in price are comparable to improvement in service in
terms of the
(A) tangibility of the benefits that they tend to confer
(B) increased revenues that they ultimately produce
(C) basis on which they need to be weighed
(D) insufficient analysis that managers devote to them
(E) degree of competitive advantage that they are likely to provide
To answer this question, first locate the question to the second sentence of the passage. "Price
reduction, like improvement in service, must be balanced against other types of efforts on the
basis of direct, tangible benefits such as increased revenues." Now, go back to answer choices.
Choice D and E are irrelevant to the original sentence, so eliminate them. Then, look at the choice
A and B, both of them repeat the original sentences.
(A)

tangibility of the benefits that they tend to confer

(B)

increased revenues that they ultimately produce

(C)

basis on which they need to be weighed

Both A and B seem to be correct. However, reduction in price is comparable to that of


improvement in service in term of the basis on direct and tangible benefits, not on the tangibility or
specific benefits of increased revenues. So neither A nor B is correct. Choice B does not repeat
the same words, but address the basis for comparison. Therefore, C is the correct answer.

Section 2: Two Styles


There is an endless number of writing techniques that authors use to present their ideas.
However, there are only two writing styles used in a GRE reading passage: presentation and
argumentation.
1. Presentation
This technique is to present an idea that the author will agree or at least partially agree. The
author strengthens his position by citing relevant evidences, each related to other in a highly
structured manner. We call this style of writing as presentation. Sometimes, the author sometime
may intentionally contrast his position with an opposing view. But most often the author is just
anticipating an objection, he will soon refute it.
Here is a sample passage in presentation.
China as a nation faces two major financial problems. First, eighty-four percent of
state-owned enterprises do not generate profit. Government failed to collect money from
such business. Rather, it has to appropriate substantial funds to these enterprises in order to
prevent them from going bankrupt and thus resulting in high unemployment rate. Second,
203 million of civilians in countryside will not be able to gain pension after they retire due to
the limited budget of government.
I would like to make an outrageous suggestion that would at one stroke generate finance
earnings and provide funds for civilians retirement. I would propose that government sells its
holdings in state-owned enterprises on the open market. Such sales would provide
substantial funds for village civilians pension. At the same, they could cut down financial
burden on these state-owned enterprises.
You might object that government would be deprived of the opportunity to share its
enterprises profit if someday they make money. I agree. Sell holdings of enterprises that
would never generate profit. But, you might reply, every enterprise that competes on the
market has potential. Here we part company. Theoretically, you may be correct in claiming
that every enterprise has the potential to make money. Practically, you are wrong.
I refer to the thousands of state-owned enterprises that are not likely to make money. These
companies are 100 percent held by the nation as a whole. Government officials are
appointed as the chairman, CEO and president. The management was not responsible for
the public interest, but for the nation as a whole. If there is no significant loss in business,
they will soon be promoted back to the higher level position in government. If their
companies perform great, these executives receive direct money compensation. However,

their salary, when combined with such compensation, will be far below that of their
counterpart in private company.
It would be unrealistic to suggest that village civilians would have sufficient funds if
governments shares were sold on the open market. But the demand for compensating the
state-own enterprises would be substantially reduced.
The author developed the above passage by first pointing out a problem, suggesting a solution,
anticipating counter-position, illustrating an example, refuting a second solution, and further
anticipating possible objections. Obviously, this writing technique is presentation.
2. Argumentation
The second writing style is argumentation. This technique has a number of variations, but the
most common and direct is to develop two to three ideas and then point out why one is better than
the other or just simply refute all of them and developed the author's own idea.
Some common tip-off sentences to this method of analysis are:
z

It was traditionally assumed...

It was once believed...

It was frequently assumed ..

It was universally accepted..

Many scientists have argued...

The passage that discusses Indian and Chinese software firms represents a typical
argumentation. At the beginning, the author presented a phenomenon and gave an explanation,
but refuted that explanation immediately.
Indian firms have achieved the highest levels of efficiency in the world software outsourcing
industry. Some researchers have assumed that Indian firms use the same programming
languages and techniques as Chinese firms but have benefited from their familiarity with
English, the language used to write software code. However, if this were true, then one
would expect software vendors in Hong Kong, where most people speak English, to perform
not worse than do Indian vendors. However, this is obviously not the case.
Then, the second explanation was introduced, but was denied again in the same paragraph.
Other researchers link high Indian productivity to higher levels of human resource investment
per engineer. But a historical perspective leads to a different conclusion. When the two top
Indian vendors matched and then doubled Chinese productivity levels in the mid-eighties,
human resource investment per employee was comparable to that of Chinese vendors.
8

Furthermore, by the late eighties, the amount of fixed assets required to develop one
software package was roughly equivalent in India and in the China. Since human resource
investment was not higher in India, it had to be other factors that led to higher productivity.
Finally, a more fruitful one is presented. The author used the remaining passage try to argue that
this explanation is the correct one.
A more fruitful explanation may lie with Indian strategic approach in outsourcing. Indian
software vendors did not simply seek outsourced contract more effectively: they made
aggressive strategic in outsourcing. For instance, most software firms of India were initially
set up to outsource the contract in western countries, such as United States. By contrary,
most Chinese firms seem to position their business in China, a promising yet
under-developed market. However, rampant piracy in China took almost 90 percents of
potential market, making it impossible for most Chinese firms to obtain sufficient
compensation for the investment on development and research, let alone thrive in
competitive environment.
Why bother to identify the writing style?
Be familiar with the author's writing techniques can help you diagram the mental road map of a
passage, identify the author's intention to cite an evidence, main idea of a passage, and most
importantly, pick up the right choice quickly and decisively. Lets go back the passage that talks
about whether price reduction can generate a competitive advantage.
The fact that reducing price can generate a competitive advantage for a company does not
mean that every reduction in price will create such an advantage. Price reduction, like
improvement in service, must be balanced against other types of efforts on the basis of direct,
tangible benefits such as increased revenues. If a company is already effectively on a par
with its competitors because it provides product at an acceptable price and keeps customers
from leaving at an unacceptable rate, then reduction in price may not be effective, since price
is not necessarily the deciding factor for any customer in any situation.
This truth was not apparent to managers of one operating system software vendor, which
failed to improve its competitive position despite its attempt to reduce price. The software
managers did not recognize the level of customer inertia that arises from the inconvenience
of switching operating system. Nor did they analyze their reduction in price to determine
whether it would attract new customers by producing a new standard of price that would
excite customers or by proving difficult for competitors to copy.
In the above passage, the author did not try to present his own position (presentation). If any, the
position is that he does not agree with the fact that reduction in price can generate competitive

advantage for a company. In fact, the speaker here argued against a popular point of view by
reasoning and examples (argumentation).
Let's look at a sample question to see how to pick up a right choice on the basis of writing styles.
The primary purpose of the passage is to
(A) contrast possible outcomes of a type of business strategy
(B) suggest more careful evaluation of a type of business strategy
(C) illustrate various ways in which a type of business strategy could fail to enhance revenues
(D) trace the general problems of a company to a certain type of business strategy
(E) criticize the way in which managers tend to analyze the costs and benefits of business
strategies
This question asks you to summarize the passage's central idea. Which of the five choices is
correct? Based on the verbs initiating the five choices, you can eliminate three of them:
(A) incorrect. To contrast is to compare several things, but not to agree or disagree.
(C) incorrect. To illustrate is to give example, not to agree or disagree.
(D) incorrect. To trace is to track, not to agree or disagree.
Choice E began with argumental word criticize, but isn't the correct choice because it addresses
the detail. Therefore, B is the right answer: to argue that superior service does not generate
competitive advantage is to suggest more careful evaluation of a type of business strategy (price
reduction).
3. Organizational Structure
There are two major patterns that the test-maker uses to reach a conclusion: general-to-specific
and specific-to-general. Become familiar with these writing patterns can help you identify the main
idea of a passage.
A. General-to-Specific Structure
This structure is widely used in GRE reading passage. The test-writer first makes a general
argument, and then supports it using a series of specific examples or reasoning, and finally
summaries by reclaiming his general argument.
Here is the structure:
z

General claim, followed by

first evidence or reasoning

second evidence or reasoning


10

more evidence or reasoning

Let's look at a passage of this structure:


The fact that reducing price can generate a competitive advantage for a company does not
mean that every reduction in price will create such an advantage. Price reduction, like
improvement in service, must be balanced against other types of efforts on the basis of direct,
tangible benefits such as increased revenues. If a company is already effectively on a par
with its competitors because it provides product at an acceptable price and keeps customers
from leaving at an unacceptable rate, then reduction in price may not be effective, since price
is not necessarily the deciding factor for any customer in any situation.
This truth was not apparent to managers of one operating system software vendor, which
failed to improve its competitive position despite its attempt to reduce price. The software
managers did not recognize the level of customer inertia that arises from the inconvenience
of switching operating system. Nor did they analyze their reduction in price to determine
whether it would attract new customers by producing a new standard of price that would
excite customers or by proving difficult for competitors to copy.
Here, the author presents his opinion at the beginning of the passage: reduction in price does not
necessarily generate a competitive advantage. To support his idea, the author first made
reasoning by comparing service improvement and price reduction. Then, in the second paragraph,
the author used an example within operating system software industry to further address that
reducing price did not improve competitive position.
B. Specific-to-General Structure
Contrast to the general-to-specific structure, the specific-to-general first presents a group of
examples or reasoning and finally draw a conclusion.
Here is the structure:
z

first example or reasoning

second example or reasoning

more example or reasoning

Conclusion

The passage that discusses Indian software vendors was written in argumentation, and
represents a typical passage in specific-to-general structure.
Indian firms have achieved the highest levels of efficiency in the world software outsourcing
industry. Some researchers have assumed that Indian firms use the same programming
languages and techniques as Chinese firms but have benefited from their familiarity with

11

English, the language used to write software code. However, if this were true, then one
would expect software vendors in Hong Kong, where most people speak English, to perform
not worse than do Indian vendors. However, this is obviously not the case.
Other researchers link high Indian productivity to higher levels of human resource investment
per engineer. But a historical perspective leads to a different conclusion. When the two top
Indian vendors matched and then doubled Chinese productivity levels in the mid-eighties,
human resource investment per employee was comparable to that of Chinese vendors.
Furthermore, by the late eighties, the amount of fixed assets required to develop one
software package was roughly equivalent in India and in the China. Since human resource
investment was not higher in India, it had to be other factors that led to higher productivity.
A more fruitful explanation may lie with Indian strategic approach in outsourcing. Indian
software vendors did not simply seek outsourced contract more effectively: they made
aggressive strategic in outsourcing. For instance, most software firms of India were initially
set up to outsource the contract in western countries, such as United States. By contrary,
most Chinese firms seem to position their business in China, a promising yet
under-developed market. However, rampant piracy in China took almost 90 percents of
potential market, making it impossible for most Chinese firms to obtain sufficient
compensation for the investment on development and research, let alone thrive in
competitive environment.
In the above passage, the author gave an explanation to a particular event, but refuted it soon,
until it came with a convincing one the conclusion.

12

Section 3: Three Subjects


Like writing techniques, GRE subjects may vary significantly. The author may present how
caffeine activates human behavior, discuss what causes Japanese auto companies to perform
better than those in USA, or explain the union's effort to organize the employees in public sectors.
Various as the subjects may be, there are only three major subjects that a GRE passage may be
discussing about: natural science, social science, and business. As the name indicates, natural
science topic includes biology, chemistry, geology, and archeology; social science includes art,
literature, and civil rights; business includes marketing, advertising, management, and
economics.
1. Natural Science
Characteristics
Most test-takers find the natural science difficult to read and beyond their knowledge base. If you
try to figure out what they are really talking about, in most cases you will fail, because reading
passages in this kind of subject are filled of nomenclatures and jargons. The good news, however,
is that the sentences are always written in a simple syntax and the questions to be answered after
reading passage are typical of the recalled questions. That means, when you successfully locate
the "key words", you will find it easy to get the right answer.
Strategy
Don't memorize the details or try to figure out the author's reasoning. Skim the passage, and get
its central idea as well as organizational structure.
The following passage is about natural science topic. This passage is a little difficult to
understand, but the following questions are much easier to answer.
Sample Passage
The cutting-edge science is ringing alarm bells. Avian flu virus picked up by pigs can swap
genetic materials with another flu virus already in the pig and become a new, hitherto
unknown flu virus for which no person, no animal has preexisting immunity. The kind of virus
causes a pandemic because it spreads from human to human.
If you took a peek into history, it turns out that previous influenza pandemics have similar
scenarios. The greatest influenza pandemic in 1918 caused more than 20 million deaths of
soldiers stationed in France. The last influenza pandemic was in 1968, known as the Hong
Kong flu (H3N2). Thousands of deaths and millions were infected worldwide.
The other examples are the Nipah virus and Japanese Encephalitis virus, which find pigs to
be good hosts. With JE, the virus circulates in the blood of infected pigs. When infected pigs

13

are bitten by Culex mosquitoes, the virus replicates in the mosquito's gut. The next time the
mosquito bites a human, the virus is passed on. The pig doesn't get sick as such. The Nipah
virus causes pneumonia symptoms in pigs. In humans, it causes encephalitis, and humans
catch it only with direct contact with infected pigs. Symptoms range from mild headache to
permanent brain damage, and can be fatal.
It's merely a phenomenon of nature that the pig is the "mixing vessel" for the new germ. But
make no mistake, the pig is not the villain, neither is the chicken. It's actually us, and our
horrible farm practices, outdated agricultural policy and, most of all, reckless disregard of our
ecology and environment. "Hygiene and management can control what eventually happens,"
says Lam. "Good farming practice will prevent serious outbreaks and infection to humans."
Despite knowing that, animal diseases and the possibility of transmission to humans are
becoming quite alarming. Of the 35 new emerging diseases in the last 20 years, more than
70 per cent involved animals.
In fact, what we may have done is unwittingly create the perfect launch pad for an influenza
pandemic that will likely kill large numbers of people across the globe. Although scientists
say it's impossible to predict the odds that the virus will alter its genetic form radically enough
to start leaping from human to human, the longer H5N1 is out there killing chickens, the
higher the chances are.
Sample Question
Which of the following statement can be inferred from the passage?
(A) New emerging diseases causes more deaths of human than animal.
(B) Animals are the villain for most flues.
(C) Hygiene and management can not control the spread of viruses.
(D) The current bird flu epidemic may be a launch pad for the next influenza pandemic.
(E) The influenza pandemic is always a regional phenomenon.
Which answer is correct? For choice A, the passage did not make any comparison between
deaths of human and deaths of animal. In B, animal is actually not the villain for most flues. Rather,
it is human. Look at the second sentence in the fourth paragraph, But make no mistake, the pig
is not the villain, neither is the chicken. For C, Hygiene and management can control what
eventually happens(in the middle of fourth paragraph), therefore, C is incorrect. E is also
incorrect. Though most flues discussed in this passage were originated from some areas, the
passage never stated it was a regional phenomenon. In fact, it will likely kill large numbers of
people across the globe, as stated at the beginning of last paragraph. The correct answer is D
the current bird flu epidemic may be a launch pad for the next influenza pandemic, because no
animal has preexisting immunity and it causes a pandemic by spreading from human to human.
14

2. Social Science
Characteristics
Why women's rights experienced a significant improvement during 1860's? How the Pullman
stroke to improve their living condition? Passages in these subjects are easy to read because it
goes as you expect and talks about something around your world. You will find it easy to grasp
the main idea and passage map. In order to get the right answer, however, you need to read
beyond the words, phrases or concepts in the passage. The right answer is always created in a
synthesized way.
Strategy
Be careful in tackling this "social" passage. To answer the later questions is always not as easy
as to understand the passage. The answer choice that contains the exact words or phrases from
the passage is generally not the correct answer. Rather, you need to synthesize several
sentences or make some reasoning before you pick up the right choice. The process is
time-consuming because the social passage is typically long.
Sample Passage
China as a nation faces two major financial problems. First, eighty-four percent of
state-owned enterprises do not generate profit. Government failed to make money from such
business. Rather, it has to appropriate substantial funds to these enterprises in order to
prevent them from going bankrupt and thus resulting in high unemployment rate. Second,
203 million of civilians in countryside will not be able to gain pension after they retire due to
the limited budget of government.
I would like to make an outrageous suggestion that would at one stroke generate finance
earnings and provide funds for civilians retirement. I would propose that government sells its
holdings in state-owned enterprises on the open market. Such sales would provide
substantial funds for village civilians pension. At the same time, they could cut down
financial burden on these state-owned enterprises.
You might object that government would be deprived of the opportunity to share its
enterprises profit if someday they make money. I agree. Sell holdings of enterprises that
would never generate profit. But, you might reply, every enterprise that competes on the
market has potential. Here we part company. Theoretically, you may be correct in claiming
that every enterprise has the potential to make money. Practically, you are wrong.
I refer to the thousands of state-owned enterprises that are not likely to make money. These
companies are 100 percent held by the nation as a whole. Government officials are
appointed as the chairman, CEO and president. The management was not responsible for

15

the public interest, but for the nation as a whole. If there is no significant loss in business,
they will soon be promoted back to the higher level position in government. If their
companies perform great, these executives receive direct money compensation. However,
their salary, when combined with such compensation, will be much less than the amount
they would earn if were in private company.
It would be unrealistic to suggest that village civilians would have sufficient funds if
governments shares were sold on the open market. But the demand for compensating the
state-own enterprises would be substantially reduced.
Sample Question
According to the passage, executives in a state-owned enterprise are motivated by
(A) direct money compensation
(B) increasing salary
(C) political outlook
(D) share option
(E) social responsibility
The passage mentioned the executives of state-owned enterprises in fourth paragraph; therefore,
we need not to consider other paragraphs when referring to the original passage. Since their
salary, when combined with such compensation, will be much less than the amount they
would earn if were in private company as stated in the last sentence, these executives are not
motivated by financial earnings. If yes, they will transfer to a private company. Therefore, they are
not motivated by direct money compensation, increasing salary, or share option. Rather, they are
concerned on their political outlook. If there is no significant loss in business, they will soon
be promoted back to the higher level position in government. Choice C is the correct answer.
For choice E, the passage never discussed the executives social responsibility.

16

3. Business Subject
Characteristics
This subject is highly welcomed since most students possess some knowledge or background in
business. But passage of this subject contains the most difficult questions in GRE Reading
Comprehension. Recall questions are few and you always have to reason before you pick up the
correct choice.
Strategy
Don't rely on your memory even if you become or have been quite familiar with its topics. There
are too many traps here. Make sure you refer to the passage when answering the questions.
Sample Passage
The fact that reducing price can generate a competitive advantage for a company does not
mean that every reduction in price will create such an advantage. Price reduction, like
improvement in service, must be balanced against other types of efforts on the basis of direct,
tangible benefits such as increased revenues. If a company is already effectively on a par
with its competitors because it provides product at an acceptable price and keeps customers
from leaving at an unacceptable rate, then reduction in price may not be effective, since price
is not necessarily the deciding factor for any customer in any situation.
This truth was not apparent to managers of one operating system software vendor, which
failed to improve its competitive position despite its attempt to reduce price. The software
managers did not recognize the level of customer inertia that arises from the inconvenience
of switching operating system. Nor did they analyze their reduction in price to determine
whether it would attract new customers by producing a new standard of price that would
excite customers or by proving difficult for competitors to copy.
Sample Question
The passage suggests which of the following about price charged by an operating system software
vendor prior to its strategy in reducing its price?
(A) It was slightly low to that of the vendors competitors.
(B) It threatened to weaken the vendors competitive position with respect to other operating
system software vendor
(C) It had already been reduced after having caused damage to the vendors reputation in the
past.
(D) It enabled the vendor to retain customers at an acceptable rate

17

(E) It needed to be reduced to attain parity with the software provided by competing vendors.
Here, the question was created in complicated clauses and itself already hard to understand. In
fact, it asks for the situation of the vendor before price reduction. Only D can be inferred from the
passage. The original passage stated that If a company is already effectively on .. keeps
customers from leaving at an unacceptable rate and This truth was not apparent to
managers of one operating system software vendor That means the vendor was able to
retain customers at an acceptable rate.

18

Section 4: Four-step Process of Reading


In the previous section we summarize three kinds of subject you will encounter in a GRE reading
passage. Now you will learn the four-step procedure to read a passage in any subject:
1.

Analyze the first paragraph.

2.

Skim the passage and get some idea of the main idea

3.

Identify the purpose of each paragraph and structure of the passage

4.

Answer the questions and don't forget to refer to the passage

1. Analyze the first paragraph.


It is essential to carefully read the first paragraph. You will get informed what the passage is
talking about, and even the main idea of the passage. There are two major reasons for you to
carefully read the first paragraph.
Fist of all, the paragraph is the main structural unit of any passage. Every paragraph is needed to
understand the whole passage or answer the question after the passage. Test-maker never
delivers a junk content. It must talk about something that relates to the central idea, and present it
as persuasively as possible. In fact, the first paragraph introduces either the position that the
author will support or the one that he/she will argue against. So, getting familiar with the
introductory paragraph will definitely help you identify the main topic.
Secondly, analyzing the fist paragraph in stead of the whole passage can save you much time. As
I said at the beginning of this chapter, GRE reading passage is dry and unfamiliar. It is highly
likely that after you read the passage, you get no ideas about what the passage is talking about. If
you go back and reread the whole passage, you will have no sufficient time to answer the
question. Analyze the first paragraph, pay attention to concepts, and then you will find it easy to
understand the subject of passage.
Below is the first paragraph of a GRE reading passage. Pay attention to concept words.
China as a nation faces two major financial problems. First, eighty-four percent of
state-owned enterprises do not generate profit. Government failed to make money from such
business. Rather, it has to appropriate substantial funds to these enterprises in order to
prevent them from going bankrupt and thus resulting in high unemployment rate. Second,
203 million of civilians in countryside will not be able to gain pension after they retire due to
the limited budget of government.
The first sentence stated that China faces two problems. Then, the author specified these two
problems using a clear and logical structure. Firstly, government did not make money from but

19

input large amount of money to its enterprises. Secondly, government has limited funds for
pension.
Now, let's summarize this paragraph and put it in our own words-- China has two problems:
financial burden and limited funds. Keep these key words (concepts) in mind, and you will find it
easy to understand the remaining passage that we'll present in next step.
2. Skim the passage and get the author's main point
Here are some strategies that will speed your reading and help you identify the author's main
points:
z

Focus on the first sentence of each paragraph

The first sentence of a paragraph is always the main point of this paragraph. Why? It confirms to
the formal writing style. If you are a management consultant, you will find it a great advantage to
use a summary at the very beginning of each section. Image when you are presenting a strategy
report which contains hundreds of pages, how could your clients catch your all of them? The only
solution is to use a highly structured presentation, and summarize your idea at the beginning of
each section. In fact, you are also doing like this in the AWA section.
By simply reading the first sentence of each paragraph, you can construct a mental road map of
the passage while not spending significant time.
z

Pay attention to the mood words

"Mood" words are those that the author uses to demonstrate his/her position to a particular event,
phenomenon, or point of view. A mood word can be positive or negative. Positive words such as
successfully, correctly and right often illustrate an idea that the author agree. And vis-a-vis, a
negative word indicates an idea that will be weakened in later passage.
The following sentences express the author's position by using positive mood words:
a) Haney's through research provides convincing field evidence that..
b) For many yeas, Benjamin Quarles' seminal account of the participation of African Americans
in the American Revolution has remained the standard work in the field.
c) Roger Rosenblatt's book successfully alters the approach taken by most previous studies.
By contrast, the following mood words are negative.
fail

ignore

overestimate

underestimate

misunderstand

misrepresent

overlook

exaggerate

sound

convincingly

successfully

correctly

20

Never ignore the counter-evidence indicators

The author uses counter-evidence words not to argue against himself, but concede certain minor
points that may weaken his argument. The counter evidence is finally refuted by further evidence.
You should keep alarm to these words since some students often mistake them as introducing
arguing against a statement.
Following are some of the most common used counter-evidence indicators:
actually

despite

admittedly

except

even though

nonetheless

nevertheless

although

however

In spite of

do

may

OK. Let's go back to the passage talking about national finance. Here are the other five
paragraphs. In order for you to skim the passage using the above three techniques, we
underlined the first sentences of each paragraph, boldfaced the mood words and italicize the
counter-evidence indicators.
I would like to make an outrageous suggestion that would at one stroke generate finance
earnings and provide funds for civilians retirement. I would propose that government sells its
holdings in state-owned enterprises on the open market. Such sales would provide
substantial funds for village civilians pension. At the same time, they could cut down
financial burden on these state-owned enterprises.
You might object that government would be deprived of the opportunity to share its
enterprises profit if someday they make money. I agree. Sell holdings of enterprises that
would never generate profit. But, you might reply, every enterprise that competes on the
market has potential. Here we part company. Theoretically, you may be correct in claiming
that every enterprise has the potential to make money. Practically, you are wrong.
I refer to the thousands of state-owned enterprises that are not likely to make money. These
companies are 100 percent held by the nation as a whole. Government officials are
appointed as the chairman, CEO and president. The management was not responsible for
the public interest, but for the nation as a whole. If there is no significant loss in business,
they will soon be promoted back to the higher level position in government. If their
companies perform great, these executives receive direct money compensation. However,
their salary, when combined with such compensation, will be much less than the amount
they would earn if were in private company.

21

It would be unrealistic to suggest that village civilians would have sufficient funds if
governments shares were sold on the open market. But the demand for compensating the
state-own enterprises would be substantially reduced.
What is the main idea of the passage? In a word, the author is to present a solution to funding the
civilian pension while benefiting the state-owned enterprises.
3. Diagram the organization of the passage
You got main idea of each paragraph. Now, its time to ask yourself why the author includes them,
what the purpose of each paragraph is, and how each paragraph relates to other. This will help
you diagram the organization of a passage, and locate the details when you answer the
questions.
Pivotal words can help you in diagramming the organization. Pivotal words are signal words or
phrases that would in advance indicate the idea of paragraphs. Below represents the most
frequently used pivotal words or sentences you will see in a reading passage.
Note: A and B represent something, while sb represents somebody.
Introduction
z

When it comes to ..., some think ...

There is a public debate today that ...

A is a common way of ..., but is it a wise one?

Recently the problem has been brought into focus.

Presenting Opinion
z

Now there is a growing awareness that...

It is time we explore the truth of ...

Nowhere in history has the issue been more visible.

Further Presenting Opinion


z

... but that is only part of the history.

Another equally important aspect is ...

A is but one of the many effects. Another is ...

Besides, other reasons are...

Anticipating Objections
z

You may reply that.

22

Admittedly, ..

It is reasonable to expect...

It is not surprising that...

Exampling
z

For example(instance),...

... such as A,B,C and so on (so forth)

A good case in point is...

A particular example for this is...

Presenting Reasons
z

There are many reasons for ...

Why .... , for one thing,...

The answer to this problem involves many factors.

Any discussion about this problem would inevitably involves ...

The first reason can be obliviously seen.

Most people would agree that...

Some people may neglect that in fact ...

Others suggest that...

Part of the explanation is ...

Comparing
z

The advantages for A for outweigh the disadvantages of...

Although A enjoys a distinct advantage ...

Indeed , A carries much weight than B when sth is concerned.

A maybe ... , but it suffers from the disadvantage that...

Transitioning
z

To understand the truth of ..., it is also important to see...

A study of ... will make this point clear

Further Anticipating Objections


z

Certainly, B has its own advantages, such as...

I do not deny that A has its own merits.


23

Conclusion
z

From what has been discussed above, we may safely draw the conclusion that ...

In summary, it is wiser ...

In short...

In step 2, you are assigned to skim the passage and get the main idea. Here, let's identify the
purpose of each paragraph for the archeology passage to better understand the passage.
(First of all, the author presented the problems)
China as a nation faces two major financial problems. First, eighty-four percent of
state-owned enterprises do not generate profit. Government failed to make money from such
business. Rather, it has to appropriate substantial funds to these enterprises in order to
prevent them from going bankrupt and thus resulting in high unemployment rate. Second,
203 million of civilians in countryside will not be able to gain pension after they retire due to
the limited budget of government.
(Then, the author suggested a solution to the problems)
I would like to make an outrageous suggestion that would at one stroke generate finance
earnings and provide funds for civilians retirement. I would propose that government sells its
holdings in state-owned enterprises on the open market. Such sales would provide
substantial funds for village civilians pension. At the same time, they could cut down
financial burden on these state-owned enterprises.
(Here, the author anticipated a possible objection)
You might object that government would be deprived of the opportunity to share its
enterprises profit if someday they make money. I agree. Sell holdings of enterprises that
would never generate profit. But, you might reply, every enterprise that competes on the
market has potential. Here we part company. Theoretically, you may be correct in claiming
that every enterprise has the potential to make money. Practically, you are wrong.
(Then, the author gave an example to deny this objection)
I refer to the thousands of state-owned enterprises that are not likely to make money. These
companies are 100 percent held by the nation as a whole. Government officials are
appointed as the chairman, CEO and president. The management was not responsible for
the public interest, but for the nation as a whole. If there is no significant loss in business,
they will soon be promoted back to the higher level position in government. If their
companies perform great, these executives receive direct money compensation. However,

24

their salary, when combined with such compensation, will be much less than the amount
they would earn if were in private company.
(Finally, the author further anticipated a possible objection)
It would be unrealistic to suggest that village civilians would have sufficient funds if
governments shares were sold on the open market. But the demand for compensating the
state-own enterprises would be substantially reduced.
Now, you are able to create a mental road map for the whole passage:
Paragraph # 1: introduced two major problems that China faces.
Paragraph # 2: suggested a solution and explained why it is effective.
Paragraph # 3: anticipated a possible objection and denied it soon.
Paragraph # 4: exemplified to argue against a position initiated in the third paragraph.
Paragraph # 5: concluded that his solution is not perfect, but really effective
By making such a road map, I bet you understand this passage quite well.
4. Tackle the questions and correspondently refer to the passage.
Now that you have grasped main idea and the organizational structure of the passage, you are
about to answer the following questions. Again, dont base on your memory. Always refer to the
original passage before you pick up a choice.
China as a nation faces two major financial problems. First, eighty-four percent of
state-owned enterprises do not generate profit. Government failed to make money from such
business. Rather, it has to appropriate substantial funds to these enterprises in order to
prevent them from going bankrupt and thus resulting in high unemployment rate. Second,
203 million of civilians in countryside will not be able to gain pension after they retire due to
the limited budget of government.
I would like to make an outrageous suggestion that would at one stroke generate finance
earnings and provide funds for civilians retirement. I would propose that government sells its
holdings in state-owned enterprises on the open market. Such sales would provide
substantial funds for village civilians pension. At the same time, they could cut down
financial burden on these state-owned enterprises.
You might object that government would be deprived of the opportunity to share its
enterprises profit if someday they make money. I agree. Sell holdings of enterprises that
would never generate profit. But, you might reply, every enterprise that competes on the

25

market has potential. Here we part company. Theoretically, you may be correct in claiming
that every enterprise has the potential to make money. Practically, you are wrong.
I refer to the thousands of state-owned enterprises that are not likely to make money. These
companies are 100 percent held by the nation as a whole. Government officials are
appointed as the chairman, CEO and president. The management was not responsible for
the public interest, but for the nation as a whole. If there is no significant loss in business,
they will soon be promoted back to the higher level position in government. If their
companies perform great, these executives receive direct money compensation. However,
their salary, when combined with such compensation, will be much less than the amount
they would earn if were in private company.
It would be unrealistic to suggest that village civilians would have sufficient funds if
governments shares were sold on the open market. But the demand for compensating the
state-own enterprises would be substantially reduced.
1. The primary purpose of the passage is to propose
(A) an alternative to manage government property
(B) a way to relieve government burden while providing funds to village civilians
(C) a way to distinguish state-owned enterprises that make money from those that do no
make money
(D) the governmental approach to evaluate state-owned enterprise executives
(E) a new system for national pension system
This question requires you to identify the primary concern of the passage as a whole. The first
paragraph introduces two major problems that China faces. The second paragraph suggests a
solution and explains why it is effective. The third anticipates a possible objection and refutes it
soon. The fourth paragraph illustrates an example to support the authors argument. In the last
paragraph, the author concludes that his solution is not perfect, but really effective. Therefore, the
correct answer is B.
2. The author implies that all of the following statements about enterprises with which government
holds 100 percent share are true EXCEPT:
(A) A market for governments share already exists.
(B) Such enterprises seldom generate profit.
(C) There is likely to be a continuing loss of such enterprises.
(D) Government officers are appointed as the executives with such enterprises.

26

(E) If the executives perform poorly, they will be demoted to lower position.
The question requires you to identify the answer choice that CANNOT be inferred from the
passage. Nothing in the passage implies that the executives will be demoted to lower position if
they perform poorly. Therefore, the best answer is E. In answering the question that contains
EXCEPT, keep alarm not to be fooled by the test maker.
3. The author implies that which of the following would occur if governments shares were sold on
the open market?
I. The shortage of retirement fund in village would eventually cease completely.
II. Current executives in state-owned enterprises are not motivated to perform better
III. Civilians in countryside would be able to seek sufficient funds from government.
(A) I only
(B) II only
(C) I and II only
(D) II and III only
(E) I, II, and III
This question asks you to identify information that is suggested rather than directly stated in the
passage. To answer it, first look for the location in the passage of the information specified in
answer choice. The last paragraph states that It would be unrealistic to suggest that village
civilians would have sufficient funds if governments shares were sold on the open market,
therefore, I is incorrect. III, which is a repeated of I, is also incorrect. Only II can be inferred from
the original passage, therefore B is the best answer.

27

Section 5: Five Types of Questions


While the techniques introduced in previous four sections speed your reading, this section is
developed to help you pick up the right choice quickly and decisively. In the following passage, we
will discuss the major question types you may encounter in real GRE test. Generally, there are
only five major types of questions. As you become familiar with the following question types, you
will gain an intuitive sense for the places from which questions are likely to be drawn. Note, the
order in which the questions are asked roughly corresponds to the order in which the main issues
are presented in the passage. Early questions should correspond to information given early in the
passage, and so on.
Of course, there are many other kinds of classification according to different criteria. Here, we
classify, by how we solve reading comprehension questions, into five based on the summary of
thousands of the previous real questions. Let's preview the five question types.
Question Types Preview
1.

2.

Main Idea Question


a)

Main Topic

b)

Tone

c)

Structure

d)

Exemplifying

Recall Question
a)

Description

b)

Listing

3.

Inference Question

4.

Critical Reasoning Question

5.

a)

Analogy

b)

Assumption/Weaken/Strengthen

Unable-to-locate Question

28

1. Main Idea Question


There are four sub-types for this kind of question: Main Topic, Tone, Structure, and Exemplifying.
Why should we incorporate them into one type of question? In answering Main Idea Question,
you should understand the organizational structure of the passage, the author tone toward a
particular point of view in the reading passage, the purpose of each paragraph and why a
particular example was illustrated. In other words, if you can determine the main topic of the
passage, you are simultaneously well informed with the structure, the intent of specific example,
and tone toward specific position.
A. Main Topic
Main idea questions test your ability to identify and understand an author's intent in a passage.
The main idea is usually stated in the first or last paragraph. Main idea questions are usually the
first questions asked. Some common main idea questions include:
z

The primary purpose of the passage is to

Which of the following titles would best describe the content of the passage?

The passage supplies information that would answer which of the following questions?

Which of the following is the principal topic of the passage?

The passage is most probably an excerpt from.

Which of the following best states the central idea of the passage?

In most cases, main idea questions are easy to solve. In most GRE passage the author's primary
purpose is to persuade the reader to accept her opinion. Occasionally, it is to describe something.
By determining the relationship of each paragraph, you come up with the main ideal at the same
time. However, the GRE writers may obscure the correct answer by surrounding it with close
answer choices that stress specifics. Eliminate these choices without hesitation on the test day.
Trap 1: The main topic will not focus on certain details in the passage. If you encounter the main
ideal question, eliminate the answer choices that describe the details.
Trap 2: Pay special attention to the "repeat" answer. Certain choices may exactly repeat some or
most words of the correct answer, but do not present the central idea, therefore, is not the correct
answer.
B. Tone Question
Tone questions ask you to identify the writer's attitude. Is the writer's feeling toward the subject
positive, negative, or neutral? The following represents some ways of the questions asked.
z

Which of the following best summarizes the author's evaluation of Bailyn's fourth
proposition?

29

The author's attitude toward the culture in most factories is best described as
(A) cautious (B) critical

(C) disinterested (D) respectful (E) adulatory

However, if you did not get a feel for the writer's attitude on the first reading, check the mood
words that he chooses. Beware of answer choices that contain extreme emotions. Remember the
passages are taken from academic journals. In the rarefied air of academic circles, strong
emotions are considered inappropriate and sophomoric. The writers want to display opinions that
are considered and reasonable, not spontaneous and off-the-wall. So if an author's tone is
negative, it may be disapproving, not snide or ridiculous. If her tone is positive, it may be
approving, not ecstatic. Or if her tone is neutral, it would be not be disinterested.
C. Organizational Structure
When you can determine the right answer for main topic, generally you have been familiar with
the organizational structure. Every passage is consisted of some paragraphs, and each single
paragraph performs some certain function to the passage as a whole, by presenting, supporting
or refuting the central idea. So, think about the purpose of each paragraph as you read through
the passage.
You may encounter one type of question concerning the main idea or purpose of some certain
paragraph. Some common questions include:
z

The last paragraph of the passage performs which of the following functions?

Which of the following best describes the organization of the second paragraph?

Which of the following best describes the organization of the passage?

Which of the following best describes the relation of the first paragraph to the passage as
a whole?

It is relatively easy to solve this kind of question for two reasons. First, however the question may
ask, it is concerning the main idea of the paragraph. If you come up with a question including
"paragraph", it definitely require you to generate that paragraph. Second, paragraph is only some
element of the completed passage. If you can generate the main topic for the whole passage of
three to four paragraphs, why aren't you able to summarize just one paragraph?
D. Exemplifying
The other type of structure question, exemplifying, tests your ability to identify the intention of
author's illustrating of something, some people, or phenomenon. In answering this question, you
need to first locate the example, and then refer to opinion preceding or accompanying the
example. The right answer is the repeat of this opinion.
z

In illustrating the example in line 13-16, the author intended to.?

The author referred to the experiment in order to.?

30

In addition to the above four types, the Main Idea Question may require you to respond in other
ways, such as:
z

From what kinds of subject could this passage be excerpted?

Which of the following topic would be preceding this passage?

This question, however, is similar to the above four, since they are based on your understanding
of the completed passage.
Sample Question #1
The cutting-edge science is ringing alarm bells. Avian flu virus picked up by pigs can swap
genetic materials with another flu virus already in the pig and become a new, hitherto
unknown flu virus for which no person, no animal has preexisting immunity. The kind of virus
causes a pandemic because it spreads from human to human.
If you took a peek into history, it turns out that previous influenza pandemics have similar
scenarios. The greatest influenza pandemic in 1918 caused more than 20 million deaths of
soldiers stationed in France. The last influenza pandemic was in 1968, known as the Hong
Kong flu (H3N2). Thousands of deaths and millions were infected worldwide.
The other examples are the Nipah virus and Japanese Encephalitis virus, which find pigs to
be good hosts. With JE, the virus circulates in the blood of infected pigs. When infected pigs
are bitten by Culex mosquitoes, the virus replicates in the mosquito's gut. The next time the
mosquito bites a human, the virus is passed on. The pig doesn't get sick as such. The Nipah
virus causes pneumonia symptoms in pigs. In humans, it causes encephalitis, and humans
catch it only with direct contact with infected pigs. Symptoms range from mild headache to
permanent brain damage, and can be fatal.
It's merely a phenomenon of nature that the pig is the "mixing vessel" for the new germ. But
make no mistake, the pig is not the villain, neither is the chicken. It's actually us, and our
horrible farm practices, outdated agricultural policy and, most of all, reckless disregard of our
ecology and environment. "Hygiene and management can control what eventually happens,"
says Lam. "Good farming practice will prevent serious outbreaks and infection to humans."
Despite knowing that, animal diseases and the possibility of transmission to humans are
becoming quite alarming. Of the 35 new emerging diseases in the last 20 years, more than
70 per cent involved animals.
In fact, what we may have done is unwittingly create the perfect launch pad for an influenza
pandemic that will likely kill large numbers of people across the globe. Although scientists
say it's impossible to predict the odds that the virus will alter its genetic form radically enough
to start leaping from human to human, the longer H5N1 is out there killing chickens, the
higher the chances are.
31

Which of the following best describes the topic of the passage?


(A) What causes the Nipah virus and Japanese Encephalitis virus to happen?
(B) Does Hong Kong flu originate from pig?
(C) From fowl to pigs to humans?
(D) Is influenza pandemic horrible?
(E) Shall we eat chicken?
This question asks you to find a title for the passage. In other word, it requires you to identify the
primary concern of the passage as a whole. The first paragraph presents a recent virus. The
second and third paragraphs describe similar influenza pandemics in history. The fourth
paragraph concludes who should be responsible for the spread of virus and what human can do
to control. The last paragraph indicates that people stimulated rather than inhibited its
promulgation. We can thus conclude the current virus will also leap to human. Furthermore, the
passage as a whole is to ring alarm bells. Therefore, C is the best answer.
Sample Question #2
Indian firms have achieved the highest levels of efficiency in the world software outsourcing
industry. Some researchers have assumed that Indian firms use the same programming
languages and techniques as Chinese firms but have benefited from their familiarity with
English, the language used to write software code. However, if this were true, then one
would expect software vendors in Hong Kong, where most people speak English, to perform
not worse than do Indian vendors. However, this is obviously not the case.
Other researchers link high Indian productivity to higher levels of human resource investment
per engineer. But a historical perspective leads to a different conclusion. When the two top
Indian vendors matched and then doubled Chinese productivity levels in the mid-eighties,
human resource investment per employee was comparable to that of Chinese vendors.
Furthermore, by the late eighties, the amount of fixed assets required to develop one
software package was roughly equivalent in India and in the China. Since human resource
investment was not higher in India, it had to be other factors that led to higher productivity.
A more fruitful explanation may lie with Indian strategic approach in outsourcing. Indian
software vendors did not simply seek outsourced contract more effectively: they made
aggressive strategic in outsourcing. For instance, most software firms of India were initially
set up to outsource the contract in western countries, such as United States. By contrary,
most Chinese firms seem to position their business in China, a promising yet
under-developed market. However, rampant piracy in China took almost 90 percents of
potential market, making it impossible for most Chinese firms to obtain sufficient

32

compensation for the investment on development and research, let alone thrive in
competitive environment.
Which of the following best describes the organization of the first paragraph?
(A) A thesis is presented and supporting examples are provided.
(B) Opposing views are presented, classified, and then reconciled.
(C) A fact is stated, and an explanation is advanced and then refuted.
(D) A theory is proposed, considered, and then amended.
(E) An opinion is presented, qualified, and then reaffirmed.
This question requires you to identify the organizational structure of the first paragraph. In this
paragraph, the author first states a fact that Indian firms achieved the highest efficiency in software
outsourcing. Then, an assumption is presented to explain such phenomenon. However, the author
refuted this explanation soon. Thus, C is the best answer.

33

2. Recall Question
There are two subtypes of recall questions: detail-locating and listing. In the following passage,
we'll discuss one by one.
A. Detail-locating
Locating question is the most common question you will encounter in Reading Comprehension. It
roughly constitutes to 50-60% of total numbers of questions. That means, in every reading
passage, there will be about one or two detail-locating questions. It is quite simple, however, to
solve this seemingly difficult question if you are able locate the detail tested. The right answer
choice is rewritten from certain sentence in the passage by changing some words or phrases. For
example, test writer will change some words from adjective to adverbial, from noun to gerund, or
just change to its synonym.
Strategy: How to locate
Below we will introduce the three-step method to locate detail.
(1) Before you locate the question to passage, you need to determine what to locate. Key words
are something that is mentioned both in the question and in passage. Then, what are key words?
Look at the following question:
Which of the following is mentioned in the passage as a disadvantage of storing artifacts
in museum basements?
Here, key words will not be any word or phrase of "which of the following is mentioned in the
passage as", but will be "disadvantage" from "disadvantage of storing artifacts in museum
basements".
We call this step as Defining Key Words.
(2) After you define key words, you are turning to the original passage. Sometimes, the key words
will appear several times in different parts of the passage. Where should you refer to? Generally,
you should locate the key words to the sentence in which key words first appear. After all, you
have only several minutes to complete a passage.
(3) When you determined which sentence (sometimes, two or more sentences) to locate, get
some idea, then quickly refer to the answer choices. Do not spend too much time analyzing this
sentence since it may be too long or complicated to understand. If the choice mentions something
that only appears in other part of the passage, eliminate it. Also eliminate the choice that just
repeats the words or phrases from original passage.
B. Listing
The other type of Recall Question is listing. As the name indicates, Listing Question requires you
to identify some people, actions, or situations that are enumerated in the passage.
34

Here are some Listing Questions:


z

According to the passage, senior managers use intuition in all of the following ways
EXCEPT to:

According to the passage, critics of the Ewha women's studies program cited the program
as a threat to which of the following?

I.

National identity

II.

National unification

III. Economic development


IV. Family integrity
(A) I only (B) I and II only (C) I, II, and III only
(D) II, III, and IV only (E) I, II, III, and IV
To solve this type of question, you should first name the key word from the stimulus, and locate it
to original passage. Then, you will find some lists that are similar to the answer choices. Carefully
compare those lists one by one to the answer choices. Use POE to eliminate incorrect choice,
until you find the right one.

35

3. Inference Questions
Inference question is the second most common. Unlike recall question, inference questions
require you to go beyond the passage. That means, the correct answer must say more than what
is said in the passage. Beware of same language traps with these questions: the correct answer
will often both paraphrase and extend a statement in the passage, but it will not directly quote it. If
you are puzzled how to determine whether a detail question is recall question or inference
question, pay attentions to the way the question asks. Generally, inference question will include
some word, such as infer, suggest and imply that indicates what kind of question it is.
z

It can be inferred from the passage that...

The passage/author suggests that.

The passage/author implies that...

Since we must not directly refer to the original passage in answering inference question, we need
to decipher the inference. Next, we will show you how to reason from couples of sentence.
Technique 1 Reasoning by Word of Comparison
The question is asking about B, but you may be unable to directly identify the characters of B
even you have located B. Rather, the original sentence is discussing about A. Here, you should
turn to the word that indicates comparison between A and B. Some words that indicate strong
comparison are unlike, in contrast to, by contrast and compared with. When you can determine
the character of B, you can simultaneously determine A is B or non-B. Also, the passage may
compare two particular events by dates or places. The phrases could be "prior to 1975" or "since
mid-1970's".
Technique 2 Reasoning by Syllogism
In logics, Syllogism looks like this: every virtue is laudable; kindness is a virtue; therefore,
kindness is laudable. As we put it in more simple way, it may be "AB and, then AC". It may be
relatively easy to recognize AB by locating the key word in the question, but it will always take
some time to identify BC, since they may be located in other part of the place. So pay attention
to the pronouns (it or they) and the nouns with definite article "the" since they often serve as B.
The fact that reducing price can generate a competitive advantage for a company does not
mean that every reduction in price will create such an advantage. Price reduction, like
improvement in service, must be balanced against other types of efforts on the basis of direct,
tangible benefits such as increased revenues. If a company is already effectively on a par
with its competitors because it provides product at an acceptable price and keeps customers
from leaving at an unacceptable rate, then reduction in price may not be effective, since price
is not necessarily the deciding factor for any customer in any situation.
36

This truth was not apparent to managers of one operating system software vendor, which
failed to improve its competitive position despite its attempt to reduce price. The software
managers did not recognize the level of customer inertia that arises from the inconvenience
of switching operating system. Nor did they analyze their reduction in price to determine
whether it would attract new customers by producing a new standard of price that would
excite customers or by proving difficult for competitors to copy.
The passage suggests which of the following about price charged by an operating system software
vendor prior to its strategy in reducing its price?
(A) It enabled the vendor to retain customers at an acceptable rate
(B) It threatened to weaken the vendors competitive position with respect to other operating
system software vendor
(C) It had already been reduced after having caused damage to the vendors reputation in the
past.
(D) It was slightly low to that of the vendors competitors.
(E) It needed to be reduced to attain parity with the software provided by competing vendors.
Here, the question asks for the situation of the vendor before price reduction. Only A can be
inferred from the passage. The original passage stated that If a company is already effectively
on .. keeps customers from leaving at an unacceptable rate and This truth was not
apparent to managers of one operating system software vendor That means the vendor was
able to retain customers at an acceptable rate.

37

4. Critical Reasoning Question


Even in reading comprehension, you will encounter some critical reasoning questions: analogy,
assumption, weaken, and strengthen. Here, the whole passage is an argument with premises,
assumptions and conclusions. The question asks you to identify the reasoning, critique the
argument or recognize the potential assumption. When you need to do is also to first locate the
conclusion to particular sentence of the passage, then identify the evidence and conclusion. The
premise (or evidence) could be near to the conclusion, or in other part of the 3-4 paragraph
passage. So, it is more difficult than its counterpart in critical reasoning section. That's why it
appears more often in high difficult level screen.
A. Analogy
Also known as application question, analogy question requires you to identify the author's
reasoning somewhere in the passage and then ask you to select one from the following five
answer choices that reasons as that is presented in the passage.
The answer choices are generally long and complicated, but they are not so difficult to
understand. After you locate the details to certain sentences in the passage, try to identify the
reasoning, and then turn to the answer choices. Fortunately, once you identify the reasoning, you
will quickly get the right answer since there are great differences among these five choices.
B. Assumption/Weaken/Strengthen
Weaken, Support, and Assumption are the other three types of question you are expected to
solve in critical reasoning question. As we said above, you need to evaluate the argument and
identify the assumptions. Typical questions would be:
z

Which of the following, if true, would most weaken the theory proposed by Snyder et al?

Which of the following, if true, would most strongly support Keyssar's findings as they are
described by the author?

Sample Question
The cutting-edge science is ringing alarm bells. Avian flu virus picked up by pigs can swap
genetic materials with another flu virus already in the pig and become a new, hitherto
unknown flu virus for which no person, no animal has preexisting immunity. The kind of virus
causes a pandemic because it spreads from human to human.
If you took a peek into history, it turns out that previous influenza pandemics have similar
scenarios. The greatest influenza pandemic in 1918 caused more than 20 million deaths of
soldiers stationed in France. The last influenza pandemic was in 1968, known as the Hong
Kong flu (H3N2). Thousands of deaths and millions were infected worldwide.
38

The other examples are the Nipah virus and Japanese Encephalitis virus, which find pigs to
be good hosts. With JE, the virus circulates in the blood of infected pigs. When infected pigs
are bitten by Culex mosquitoes, the virus replicates in the mosquito's gut. The next time the
mosquito bites a human, the virus is passed on. The pig doesn't get sick as such. The Nipah
virus causes pneumonia symptoms in pigs. In humans, it causes encephalitis, and humans
catch it only with direct contact with infected pigs. Symptoms range from mild headache to
permanent brain damage, and can be fatal.
It's merely a phenomenon of nature that the pig is the "mixing vessel" for the new germ. But
make no mistake, the pig is not the villain, neither is the chicken. It's actually us, and our
horrible farm practices, outdated agricultural policy and, most of all, reckless disregard of our
ecology and environment. "Hygiene and management can control what eventually happens,"
says Lam. "Good farming practice will prevent serious outbreaks and infection to humans."
Despite knowing that, animal diseases and the possibility of transmission to humans are
becoming quite alarming. Of the 35 new emerging diseases in the last 20 years, more than
70 per cent involved animals.
In fact, what we may have done is unwittingly create the perfect launch pad for an influenza
pandemic that will likely kill large numbers of people across the globe. Although scientists
say it's impossible to predict the odds that the virus will alter its genetic form radically enough
to start leaping from human to human, the longer H5N1 is out there killing chickens, the
higher the chances are.
All of the following situations are similar to the spread of avian flu virus described in the first
paragraph EXCEPT:
(A) The BT2 spread from a pig to another pig, and thus causes significant disease in pig.
(B) The AIDS viruses transferred from monkeys to man and spread across the world.
(C) The SARS virus originates from some wildlife and is picked up by civet cats from which
humans got it.
(D) Nipah virus circulates in the blood of infected pig, which is bitten by Culex mosquitoes, the
virus replicates in the mosquito's gut. The next time the mosquito bites a human, the virus
is passed on.
(E) H5N1 starts in chickens and leaps from human to human.
The question requires you to recognize a situation that is not similar to the spear of avian flu.
Before considering following answer choices, we fist define its rationale. It is something like this:
Avian flu virus picked up by pigs and is transferred to human. All of the situations described in the
answer choices are similar to it ex that in choice A (from animal to animal). Therefore, A is the
best answer.

39

5. Difficult-to-locate Question
Some question does not ask for the central idea of a passage. Rather, it requires you to draw a
conclusion based on the passage:
z

According to the passage, which of the following is the author most likely to agree with?

The passage supplies information that would answer which of the following questions?

Unlike Recall Question or Inference Question, Difficult-to-locate Question does not contain key
words that you can use to locate the details tested. In order to solve this type of question, you
have to skim through the passage again and again until you get the right answer. Eliminating
wrong choices often take considerable time since the answer choices are often too long and
complicated to understand. That is why most test takers regard difficult-to-locate question as the
most difficult one in reading comprehension. The good news is that if you encounter several
questions like these, then you probably get a high score since questions are presented based on
your performance on the previous questions.
Sample Question
Indian firms have achieved the highest levels of efficiency in the world software outsourcing
industry. Some researchers have assumed that Indian firms use the same programming
languages and techniques as Chinese firms but have benefited from their familiarity with
English, the language used to write software code. However, if this were true, then one
would expect software vendors in Hong Kong, where most people speak English, to perform
not worse than do Indian vendors. However, this is obviously not the case.
Other researchers link high Indian productivity to higher levels of human resource investment
per engineer. But a historical perspective leads to a different conclusion. When the two top
Indian vendors matched and then doubled Chinese productivity levels in the mid-eighties,
human resource investment per employee was comparable to that of Chinese vendors.
Furthermore, by the late eighties, the amount of fixed assets required to develop one
software package was roughly equivalent in India and in the China. Since human resource
investment was not higher in India, it had to be other factors that led to higher productivity.
A more fruitful explanation may lie with Indian strategic approach in outsourcing. Indian
software vendors did not simply seek outsourced contract more effectively: they made
aggressive strategic in outsourcing. For instance, most software firms of India were initially
set up to outsource the contract in western countries, such as United States. By contrary,
most Chinese firms seem to position their business in China, a promising yet
under-developed market. However, rampant piracy in China took almost 90 percents of
potential market, making it impossible for most Chinese firms to obtain sufficient
40

compensation for the investment on development and research, let alone thrive in
competitive environment.
According to the passage, which of the following statements is true of Indian software developers?
(A) Their productivity levels did not equal those of Chinese software engineers until the late
eighties.
(B) Their high efficiency levels are a direct result of English language familiarity.
(C) They develop component-specific software.
(D) They are built to outsource the western orders.
(E) They develop more packages of software than do those in Chinese developers.
In the middle of the last paragraph, the author states that For instance, most software firms of
India were initially set up to outsource the contract in western countries, such as United
States. Thus, the best answer is D.

41

Section 6: Six test points


While four-step procedure helps you to understand a passage and the five types of question
guide you how ETS test the understanding of the passage, the six test points will in advance
introduce what would be tested even before you read the questions. As you are reading the
passage, keep alarm to certain words or phrases since they would later act as clues for
answering the following questions. We call these signal words or phrases as test points.
In the following passage, we will introduce you the six most common test points in reading
comprehension. Once you become familiar with these test points, you will get advantage in speed
to come up with the right answer choice.
1. Comparison
Words or phrases: like, unlike, in contrast to, similarly
Question Type: recall question, inference question
Here is an example:
The fact that reducing price can generate a competitive advantage for a company does not
mean that every reduction in price will create such an advantage. Price reduction, like
improvement in service, must be balanced against other types of efforts on the basis of
direct, tangible benefits such as increased revenues. If a company is already effectively on a
par with its competitors because it provides product at an acceptable price and keeps
customers from leaving at an unacceptable rate, then reduction in price may not be effective,
since price is not necessarily the deciding factor for any customer in any situation.
This truth was not apparent to managers of one operating system software vendor, which
failed to improve its competitive position despite its attempt to reduce price. The software
managers did not recognize the level of customer inertia that arises from the inconvenience
of switching operating system. Nor did they analyze their reduction in price to determine
whether it would attract new customers by producing a new standard of price that would
excite customers or by proving difficult for competitors to copy.
Sample question
According to the passage, reduction in price are comparable to improvement in service in
terms of the
(A) tangibility of the benefits that they tend to confer
(B) increased revenues that they ultimately produce

42

(C) basis on which they need to be weighed


(D) insufficient analysis that managers devote to them
(E) degree of competitive advantage that they are likely to provide
To answer this question, first locate the question to the second sentence of the passage. "Price
reduction, like improvement in service, must be balanced against other types of efforts on the
basis of direct, tangible benefits such as increased revenues." In other words, they are
comparable based on which they need to be weighed. Therefore, C is the correct answer.

43

2. Example & Listing


Words or phrase: such as, as well as, for example, for instance
Question type: Listing, Exampling
Let's look at a sample question for the same passage.
The fact that reducing price can generate a competitive advantage for a company does not
mean that every reduction in price will create such an advantage. Price reduction, like
improvement in service, must be balanced against other types of efforts on the basis of direct,
tangible benefits such as increased revenues. If a company is already effectively on a par
with its competitors because it provides product at an acceptable price and keeps customers
from leaving at an unacceptable rate, then reduction in price may not be effective, since price
is not necessarily the deciding factor for any customer in any situation.
This truth was not apparent to managers of one operating system software vendor,
which failed to improve its competitive position despite its attempt to reduce price. The
software managers did not recognize the level of customer inertia that arises from the
inconvenience of switching operating system. Nor did they analyze their reduction in price to
determine whether it would attract new customers by producing a new standard of price that
would excite customers or by proving difficult for competitors to copy.
The discussion of the operating system software vendor last paragraph serves which of the
following functions within the passage as a whole?
(A) It describes an exceptional case in which reduction in price actually failed to produce a
competitive advantage.
(B) It illustrates the pitfalls of choosing to reduce price at a time when business strategy is
needed more urgently in another area.
(C) It demonstrates the kind of analysis that managers apply when they choose one kind of
business strategy over another
(D) It supports the argument that strategies in certain aspects are more advantageous than
strategies in other aspects.
(E) It provides an example of the point about reduction in price made in the first paragraph.
Clearly, the author intends to prove his position that reduction in price does not necessarily
generate competitive advantage. E is the correct.

44

3. People, Date & Place


Phrase: in the nineteenth-century, prior to mid-1970's, Snyder proposed that.
Question: inference question, main idea question
Indian firms have achieved the highest levels of efficiency in the world software outsourcing
industry. Some researchers have assumed that Indian firms use the same programming
languages and techniques as Chinese firms but have benefited from their familiarity with
English, the language used to write software code. However, if this were true, then one
would expect software vendors in Hong Kong, where most people speak English, to perform
not worse than do Indian vendors. However, this is obviously not the case.
Other researchers link high Indian productivity to higher levels of human resource investment
per engineer. But a historical perspective leads to a different conclusion. When the two top
Indian vendors matched and then doubled Chinese productivity levels in the mid-eighties,
human resource investment per employee was comparable to that of Chinese vendors.
Furthermore, by the late eighties, the amount of fixed assets required to develop one
software package was roughly equivalent in India and in the China. Since human resource
investment was not higher in India, it had to be other factors that led to higher productivity.
A more fruitful explanation may lie with Indian strategic approach in outsourcing. Indian
software vendors did not simply seek outsourced contract more effectively: they made
aggressive strategic in outsourcing. For instance, most software firms of India were initially
set up to outsource the contract in western countries, such as United States. By contrary,
most Chinese firms seem to position their business in China, a promising yet
under-developed market. However, rampant piracy in China took almost 90 percents of
potential market, making it impossible for most Chinese firms to obtain sufficient
compensation for the investment on development and research, let alone thrive in
competitive environment.
The author suggests that if the researchers of India mentioned in paragraph 1 were correct, which
of the following would be the case?
(A) The computer used in India software firms would be different from the computer used in
China firms.
(B) Indian engineers would be trained to do several different programming jobs.
(C) Familiarity with English language would not have an influence on the productivity levels of
engineers.
(D) The engineers in India-run firms would have lower productivity levels if they have a poor
command of English.
45

(E) The production levels of India-run firms located in the China would be equal to those of
firms run by China firms.
If the researchers are correct, then the familiarity with English determines the productivity of
engineers. That is, if the engineers in India-run firms have a poor command of English they would
have lower productivity levels, as stated in choice D.

46

4. Words of Attitude and Transition


Word: Correctly, qualified, do (does, did), may (might), correctly, first (second, third)
Question Type: Tone Question, Main Topic Question, Structure Question
The fact that reducing price can generate a competitive advantage for a company does not
mean that every reduction in price will create such an advantage. Price reduction, like
improvement in service, must be balanced against other types of efforts on the basis of direct,
tangible benefits such as increased revenues. If a company is already effectively on a par
with its competitors because it provides product at an acceptable price and keeps customers
from leaving at an unacceptable rate, then reduction in price may not be effective, since price
is not necessarily the deciding factor for any customer in any situation.
This truth was not apparent to managers of one operating system software vendor, which
failed to improve its competitive position despite its attempt to reduce price. The software
managers did not recognize the level of customer inertia that arises from the inconvenience
of switching operating system. Nor did they analyze their reduction in price to determine
whether it would attract new customers by producing a new standard of price that would
excite customers or by proving difficult for competitors to copy.
The passage suggests that operating system software managers failed to consider whether or not
the price reduction mentioned last sentence
(A) was too complicated to be easily described to prospective customers
(B) made a measurable change in the experiences of customers purchasing
(C) could be sustained if the number of customers increased significantly
(D) was an innovation that competing vendors could have imitated
(E) was adequate to bring the vendors general level of price to a level that was comparable
with that of its competitors
The passage following failed to describes the failure. The best choice is D, which is stated in the
last sentence.

47

5. Counter-evidence Indicators
Counter-evidence words warn that the author is about to either make a U-turn or introduce a
counter-premise (concession to a minor point that weakens the argument).
But

Although

However

Yet

Despite

Nevertheless

Nonetheless

Except

In contrast

Even though

Counter-evidence words mark natural places for questions to be drawn. At a pivotal word, the
author changes direction. The GRE writers form questions at these junctures to test whether you
turned with the author or you continued to go straight. Rarely do the GRE writers let a pivotal word
pass without drawing a question from its sentence. As you read a passage, note the pivotal
words and refer to them when answering the questions.
Example
China as a nation faces two major financial problems. First, eighty-four percent of
state-owned enterprises do not generate profit. Government failed to make money from such
business. Rather, it has to appropriate substantial funds to these enterprises in order to
prevent them from going bankrupt and thus resulting in high unemployment rate. Second,
203 million of civilians in countryside will not be able to gain pension after they retire due to
the limited budget of government.
I would like to make an outrageous suggestion that would at one stroke generate finance
earnings and provide funds for civilians retirement. I would propose that government sells its
holdings in state-owned enterprises on the open market. Such sales would provide
substantial funds for village civilians pension. At the same time, they could cut down
financial burden on these state-owned enterprises.
You might object that government would be deprived of the opportunity to share its
enterprises profit if someday they make money. I agree. Sell holdings of enterprises that
would never generate profit. But, you might reply, every enterprise that competes on the
market has potential. Here we part company. Theoretically, you may be correct in claiming
that every enterprise has the potential to make money. Practically, you are wrong.
I refer to the thousands of state-owned enterprises that are not likely to make money. These
companies are 100 percent held by the nation as a whole. Government officials are
appointed as the chairman, CEO and president. The management was not responsible for
the public interest, but for the nation as a whole. If there is no significant loss in business,
48

they will soon be promoted back to the higher level position in government. If their
companies perform great, these executives receive direct money compensation. However,
their salary, when combined with such compensation, will be much less than the amount
they would earn if were in private company.
It would be unrealistic to suggest that village civilians would have sufficient funds if
governments shares were sold on the open market. But the demand for compensating the
state-own enterprises would be substantially reduced.
The author anticipates which of the following initial objections to the adoption of his proposal?
(A) Government will not be able to sell its holdings with state-owned enterprise.
(B) The ability of governments to control the national economy will be weakened if
state-owned enterprises are sold to private owners.
(C) It is impossible to find enterprises that will never generate profit.
(D) The poor performance of state-owned enterprises will continue.
(E) The countryside civilians are sill unable to seek financial support from government.
In the third paragraph, the author illustrates a possible objection to his proposal. The opposite
views are government would be deprived of the opportunity to share its enterprises profit
if someday they make money and every enterprise that competes on the market has
potential. Choice C presents one of them, and is the correct answer.

49

6. Special Punctuation
Punctuation: Quotation, Parentheses, Dash
Question type: Recall question, Inference Question
The cutting-edge science is ringing alarm bells. Avian flu virus picked up by pigs can swap
genetic materials with another flu virus already in the pig and become a new, hitherto
unknown flu virus for which no person, no animal has preexisting immunity. The kind of virus
causes a pandemic because it spreads from human to human.
If you took a peek into history, it turns out that previous influenza pandemics have similar
scenarios. The greatest influenza pandemic in 1918 caused more than 20 million deaths of
soldiers stationed in France. The last influenza pandemic was in 1968, known as the Hong
Kong flu (H3N2). Thousands of deaths and millions were infected worldwide.
The other examples are the Nipah virus and Japanese Encephalitis virus, which find pigs to
be good hosts. With JE, the virus circulates in the blood of infected pigs. When infected pigs
are bitten by Culex mosquitoes, the virus replicates in the mosquito's gut. The next time the
mosquito bites a human, the virus is passed on. The pig doesn't get sick as such. The Nipah
virus causes pneumonia symptoms in pigs. In humans, it causes encephalitis, and humans
catch it only with direct contact with infected pigs. Symptoms range from mild headache to
permanent brain damage, and can be fatal.
It's merely a phenomenon of nature that the pig is the "mixing vessel" for the new germ. But
make no mistake, the pig is not the villain, neither is the chicken. It's actually us, and our
horrible farm practices, outdated agricultural policy and, most of all, reckless disregard of our
ecology and environment. "Hygiene and management can control what eventually happens,"
says Lam. "Good farming practice will prevent serious outbreaks and infection to humans."
Despite knowing that, animal diseases and the possibility of transmission to humans are
becoming quite alarming. Of the 35 new emerging diseases in the last 20 years, more than
70 per cent involved animals.
In fact, what we may have done is unwittingly create the perfect launch pad for an influenza
pandemic that will likely kill large numbers of people across the globe. Although scientists
say it's impossible to predict the odds that the virus will alter its genetic form radically enough
to start leaping from human to human, the longer H5N1 is out there killing chickens, the
higher the chances are.
What does the author mean by describing the pig as mixing vessel?
(A) Pig is the place where various viruses reside.

50

(B) Pig is the pot in which viruses swap genes and become new, deadly germs.
(C) Viruses are mixed inside the body of pig.
(D) New germs come to the body of pig and reside there.
(E) Pig attracts viruses.
The question requires you to determine the meanings of mixing vessel. At the beginning of the
passage, the author states that Avian flu virus picked up by pigs can swap genetic materials with
another flu virus already in the pig and become a new, hitherto unknown flu virus for which
no person, no animal has preexisting immunity. The kind of virus causes a pandemic because it
spreads from human to human. In other words, pig is the pot in which viruses swap genes and
become new, deadly germs. Therefore, the correct answer is B.

Review
One principal
Don't rely on memory or on daily life experiences. Answer all the questions on the basis of what is
stated or implied in the passage.
Two Styles
Presentation is to deliver an idea that the author will agree or partially agree. Argumentation is to
develop two ideas or systems and then point out why one is better than the other (contrasting) or
just simply refute both of them and developed the author's own idea.
Three Subjects
There are three major subjects that a typical GRE reading passage may discuss about: natural
science, social science and business subject.
Four-step Procedure
Use the four-step process to read a passage. First dissect the first paragraph, then skim the
passage, get the main idea and diagram the organization of the passage.
Five Types of Question
There are five types of question you may encounter in real GRE test: main idea question, recall
question, inference question, critical reasoning question and the difficult-to-locate question.
Six Test Points
The most common six test points are comparison, exemplifying, people, special punctuation,
counter-evidence and mood words.

51

Chapter 2 Sentence Completion


As its name suggests, the Sentence Completion question requires you to complete a sentence
with one or two blanks. Immediately after the question are the five answer choices. Which choice
is correct solely depends on the information provided by the original sentence. Thats to say, the
original sentence is the only place where you can seek clues for filling the blank.
There are about 6 Sentence Completion questions on the test day, and in most cases, the first
question in the verbal section is Sentence Completion. Sentence Completion is rarely difficult for
most students, but sometimes it may be a problem for international students when they are
unfamiliar with the key word or phrase that serves the clue for completing sentence. You should
know there are no more clues that you can seek elsewhere like the long passage in reading
comprehension question. Thats why some international students prefer reading compression
questions to sentence completion questions. There is no shortcut for understanding the meaning
of key words. The only way is to memorize as more English words as possible.
Given that you know the meaning of every word or phrase in the question, the sentence can be
easily completed by identifying the clues in the original sentences. In the following passages, we
will discuss how to seek problem solving clues, such as transitional words that indicate the
relationship of each sub-sentence, and so on.

52

Section 1: The four types of relationship


A. Contrast
Contrast is the most common relationship in Sentence Completion. Key words indicating contrast
relationship include:

(even) though

although

but

however

nevertheless

whereas

yet

while

unlike

do(es)

otherwise

rather

notwithstanding

despite

did

whatever

in contrast

on the contrary

on the other hand

in spite of

curiously

ironically

paradoxically

surprisingly

Clue behind the relationship is that the statement before the contrast key words should be contrast
to the statement after the contrast key words.
Example #1 (One Blank)
It is not surprising that superior service can generate competitive advantage for a company, but the
effort taken to improve service can often be -----------.
(A) difficult
(B) expected
(C) diversified
(D) unpredictable
(E) promising
The missing word describes how the effort would be to improve service and the word but
suggests that the word fills the blank must contrast with the idea of not surprising. Therefore, D is
the best answer.
Example #2 (Two Blanks)
Ancient cities were----destroyed, but archeologists have found sufficient information to
demonstrate an occasionally----but generally complete picture in Tan Era.
(A) mostly.. fragmentary
(B) obviously.. necessary
(C) unwittingly.. whole
(D) partially.. famous

53

(E) fully.. necessary


The first missing word describes how ancient cities were destroyed. The phrase sufficient
information indicates that not all destroyed, and the word but indicates that the cities were
destroyed in a way that would not lead you to find sufficient information. The second missing word
describes a picture in Tan Era. The word but suggests that the word that fills the second blank
must contrast with the idea of full. Therefore, the best answer is A, which describes the ancient
cities as mostly destroyed.

54

B. Cause and Effect


The second most common is the relationship of cause and effect. That is, the situation before the
causal indicator causes the situation after the causal indicator, or vice versa. Typical key words
that indicate cause and effect include:
because

since

for

if

thus

hence

therefore

in that

consequently

result in

so that

so...that...

as a result

result from

lead to

so...as to...

accordingly

cause

give rise to

derive from

be due to

Example #1 (One Blank)


Since the primary criterion to evaluate a company is its current performance, analysts ------ to
consider its previous credit.
(A) refused
(B) wished
(C) planned
(D) extended
(E) caused
The word since indicates a cause-effect relation between the first part of sentence and the
second part. Therefore, the word that fills the blank should be not to consider its previous credit.
Choice A, which clearly states redundancy (refused), is the best answer.
Example #2 (Two Blanks)
People seem to recall the brand of an advertised product, considering most favorably the most
frequently advertised product; therefore, an advertisement that is----in newspaper advertisement
would be expected to be very----, but, actually, it is not.
(A) dominant.. impressive
(B) frequent.. bad
(C) unusual.. effective
(D) new.. rare
(E) widespread.. ineffective

55

The first part of the sentence is a statement: the more frequent an advertisement is, the more
favorable it is considered. The word therefore suggests that the missing words must, together,
make a statement in keeping with this assumption. A is the correct answer. An advertisement that
is dominant is very frequent. Because, according to the statement, the most frequent
advertisement is likely to be most favorably considered, one would expect a dominant
advertisement to be very impressive.

56

C. Explanation
The third one is explanation. Sentences of this relationship usually have two statements, one is
general, and the other is specific. The specific sentence serves to clarify or explain the general
sentence. The most commonly used key words include:

for instance

as an example

for example

in other words

that is

namely

such as

specifically

Clue in this relationship is that synonyms are used before and after the explanation key words.
Example #1 (One Blank)
According to Maslows theory of need hierarchy, material is the ---- demand of human beings, in
that it provides the founding floor from which the other demands are generated.
(A) essential
(B) basic
(C) final
(D) cheap
(E) emotional
The phrase in that suggests that the cause before in that should be similar in meaning to that
after in that. Thus, the word that fills in the blank must be a synonym to founding. B is the best
answer. The word basic means the same as founding.
Example #2 (Two Blanks)
To be a true leader, a manager must not be too----: any effective leader depends on the ability of
other people to----with each other.
(A) popular.. agree
(B) adventurous ..communicate
(C) independent.. cooperate
(D) self-confident.. argue
(E) pragmatic.. disagree
The first missing word describes a characteristic that a manager with leadership ability (a leader)
can have, but cannot have in excess. The second missing word indicates what followers must be
able to do with each other if the leader is to be effective. The colon (:) indicates that the second
part of the sentence explains or amplifies what is said in the first part. Therefore, whatever
characteristic the leader has must enable others to do with each other. C is the best answer. If a

57

manager were too independent, it would be difficult for the leaders followers to cooperate with
each other.

58

D. Similarity
In these sentences, the author compares two similar things. Typical key words include:

in the same way

just as

like

resembles

same

as

similarly

likewise

Clue behind the relationship is that the choice you pick up should make the two things of the
sentence to be similar in meaning.
Example #1 (One Blank)
Just as human beings who depend on each other, there are no ------- foliages.
(A) neglectable
(B) existing
(C) conventional
(D) dependable
(E) solitary
The phrase just as means a similar way. Since human beings depend on each other, the foliages
should also depend on each other, or are not solitary. Therefore, the best answer is E.
Example #2 (Two Blanks)
Unlike birds that simply spread viruses from an animal to another animal, pigs are the "----vessel"
where viruses swap genes and become----, deadly germs.
(A) comprehensive.. small
(B) complicated.. general
(C) frustrated.. important
(D) mixing.. new
(E) lethal.. big
The word unlike suggests that pigs do not simply spread viruses from an animal to another
animal. Rather, inside pigs, viruses swap genes. Choice D really makes sense. A place where
viruses swap genes is a mixing vessel, and thus generate new germs.

59

Section 2: Five-Step Procedure to Complete a Sentence


Step 1: Read through the whole sentence.
Read through the whole sentence and get rough understanding of the original sentence. Benefits
from reading through the whole sentence are 1) to figure out the main idea of the sentence, 2) to
get sense of the relationship between sentences, and 3) to make sure never to overlook any clue.
Step 2: Identify the key word
Identify the key word and determine the relationship of sentences. This is one of the major steps in
solving a sentence completion question. Since the key word determines the relationship between
sentences before and after key word, it serves the clue to pick up the right answer choice. Are they
contrasted, similar, explanation or casual?
Step 3: Think of a word that can fill the blank
Before you look at the answer choices, never forget to think of a word of your own that can make
sense for the blank. If there are two blanks, think of a word that would make sense for each one.
The process of recognizing a possible answer can help better understand the meanings, and
relationship of the sentences.
Step 4: Pick up an answer choice
Now, it is time to read the answer choices, and pick up the choice similar to the one you guess.
Step 5(optional): Eliminate other four choices
This step is optional since it takes time. If you think you wont get sufficient time to answer all other
verbal questions, just disregard this process. However, this process really helps. The benefit is
obvious if you are able to eliminate other four choices, you can verify your decision on a
particular choice. Furthermore, if you encounter two answer choices that are similar or exactly
contrasted, you should reread the sentence to see whether the logic follows when you substitute
that word into the blank.

60

Chapter 3 Analogy
Introduction
In an analogy question, you are given a pair of capitalized words and other five pairs of lower-case
words. You are then required to choose a pair of words among the five that has the same logical
relationship as the original capitalized pair. The analogy question not only tests your vocabulary
base, but also tests your ability to recognize the logical relationship between two words. Here is an
example.
Example
HORSE : MARE ::
(A) cat : kitten
(B) human : woman
(C) bull : cow
(D) child : adult
(E) animal : pig
The first colon (:) means "to" and the two colons (::) means "is as". We read the question as "horse
to mare is as" Next, we need to define the relationship between this pair of words. What is the
relationship between horse and mare? A mare is a female horse. So, a rationale for this analogy
could be Y (a mare) is a female X. Because analogy problems require us to look for a pair of
words that have the same relationship has the initial two words, we are looking for a pair in which
the second word is a female of second word. Once you have determined the relationship between
the given pair of words and state it in your mind in sentence form, read through the answer choices
substituting the possible pairs into the same sentence you have created to describe the initial pair.
Let's work through the answer choices. Is a kitten a female cat? No, this does not make sense. Is a
woman a female adult? Yes, this really makes sense. Choices B, C, and D do not have such
relationship.
Trap: It is extremely important to pick up the pair of words that has the same order in relationship.

61

How to define the relationship


Think of a sentence that expresses the relationship between the two capitalized words.
Your sentence should not be too general. If it is, then it is highly possible that more one answer
choices would fit into that sentence. Therefore, you should explain the relationship as precisely as
you could. The more precise your sentence, the easier it is to pick up the right answer.
Let's look at an example.
TELESCOPE : ASTRONOMER ::
(A) picture : artist
(B) environment : ecologist
(C) element : chemist
(D) brush : painter
(E) movie : director
We are trying to create a sentence that describes a relationship between the words telescope and
astronomer. Let's say we use the sentence, a telescope is an instrument commonly used in the
work of an astronomer. Therefore, a rationale for this analogy could be X (a telescope) is an
instrument commonly used in or associated with the work of a person called a Y
(astronomer). A brush is an instrument commonly used in the work of a painter. Therefore, D
is the best answer.
Dont pick up the choice until you believe it is the best of the five pairs. Lets look at an
example to see why we should approach the analogy problem in such way.
MATRIX : NUMBER ::
(A) gas : molecule
(B) snow : precipitation
(C) act : opera
(D) school : fish
(E) crystal : atom
You may define the relationship of the word matrix and number as: matrix is composed of
numerous numbers. In this way, choice A would be great because gas is composed of
numerous molecules. If you do not move to the last choice, you will not find a better pair. Choice
E is the one. Crystal is defined as a regular arrangement of atoms. For the setup pair, matrix can
be defined as a regular arrangement of numbers. The two pairs of words perfect match in
relationship. Therefore, E is the best answer.

62

Eighteen common types of relationship


There are numerous types of relationship between two words. However, the following 18 consists
to 95% of all types of relationship tested in previous GRE analogy questions.
1. Synonyms
The two capitalized words have exactly the same meaning. The correct pair of words should also
have exactly the same meanings.
Adjective to adjective
MARTIAL : MILITARY ::
(A) mysterious : runic
(B) tortuous : straightforward
(C) objective : subjective
(D) clear : complicated
(E) imprudent : damaged
The best answer is A.
Similar pairs: aphoristic : terse :: hyperbolic : exaggerated.
Adjective to noun
HEADSTRONG : WILLFULNESS ::
(A) engrossing : obliviousness
(B) fawning : subservience
(C) venerable : renown
(D) bold : tip
(E) critical : confidence
The best answer is B.
Similar pairs: lavish : extravagance :: generous : liberality :: endemic : region :: inborn : individual ::
pertinent : relevance :: redundant : superfluity :: chary : caution :: imperturbable : composure ::
despotic : tyranny.
Adjective to verb
SIMULTANEOUS : COINCIDE ::
(A) gracious : significance
(B) fast : acceleration
(C) lavish : squander
(D) intriguing : project
(E) provocative : tradition

63

The best answer is C.


Similar pairs: didactic : instruct :: comic : amuse :: indistinct : mutter :: monotonous : drone ::
contiguous : abut.
Verb to adverb
ARTICULATE : CLEARLY ::
(A) drink : quickly
(B) run : slowly
(C) act : generally
(D) shout : loudly
(E) village : remotely
The best answer is D.
2. Antonyms
The two capitalized words have the opposite meanings. The correct pair of words should also
have the opposite meanings.
Adjective to adjective
INVINCIBLE : SUBDUED ::
(A) impervious : damaged
(B) persuasive : convinced
(C) impossible : taken
(D) invisible : overlooked
(E) despicable : contented
The best answer is A.
Similar pairs: wary : gulled :: untenable : defended :: invulnerable : injured :: inscrutable :
understood and incorrigible : reformed.
Adjective to noun
AUDACIOUS : TREPIDATION ::
(A) covetous : ostentatious
(B) laconic : volubility
(C) reckless : impression
(D) equivocal : imagination
(E) transparent : permissiveness
The best answer is B.
Similar pairs: exorbitant : moderation :: illusory : reality :: perfidious : loyalty :: offensive : tact ::
paradigmatic : anomaly :: superfluous : terseness :: legitimated : claim :: confirmed : hypothesis ::
maladroit : skill :: glib : profundity :: boundless : limit :: impeccable : flaw :: specious : genuineness ::
64

truculent : gentleness :: unregenerate : remorse :: frank : secretiveness :: callow : maturity.


Adjective to verb
RETICENT : SPEAK ::
(A) annoyed : disclose
(B) snow : precipitation
(C) parsimonious : spend
(D) outmoded : show
(E) allegiant : direct
The best answer is C.
Similar pairs: flip : respect :: foolproof : fail :: airtight : leak :: fearless : daunt :: articulate : unclear ::
elaborate : sketchy :: nonchalant : concern :: taciturn : chatter :: magnanimous : begrudge.
Adverb to noun
INDOLENTLY : VENERATION ::
(A) sympathetically : verification
(B) passively : distinctness
(C) predictably : admiration
(D) perfunctorily : inspiration
(E) beneficially : lose
The best answer is D.
Verb to verb
WAFT : PLUMMET ::
(A) cut : concur
(B) snow : precipitate
(C) act : direct
(D) violate : contradict
(E) meander : dash
The best answer is E.
Similar pairs: disperse : reconvene :: dismiss : reinstate
3. Degree of intensity
In the types of analogy, the two words are similar in meaning, but one is more intense than the
other. A rationale for this analogy could be something properly described as X is Y to a very high
degree.
Adjective to adjective
GLARING : BRIGHT ::
65

(A) deafening : loud


(B) flexible : open
(C) boisterous : quiet
(D) disputatious : offhand
(E) suspicious : amused
The best answer is A.
Similar pairs: minuscule : small :: saturated : moist :: obvious : perceptible :: crucial : relevant ::
agog : interested :: fanatic : devoted. Rapacious : covetous :: prude : proper :: paranoid :
suspicious :: idolatrous : devoted :: zealous : enthusiastic :: stygian : dark :: abysmal : low.
Also :: one word is good and the other is bad.
ATTENTIVE : OFFICIOUS ::
(A) mysterious : runic
(B) refined : snobbish
(C) metamorphose : unaltered
(D) boisterous : frightened
(E) disapproving : laudatory
The best answer is B.
Similar pairs: receptive : gullible :: frugal : penurious :: compliant : obsequious :: talkative :
garrulous :: sweet : cloying :: confident : arrogant.
Verb to verb
COMPLAIN : CARP ::
(A) recommend : suggest
(B) demonstrate : indicate
(C) indulge : mollycoddle
(D) complete : finish
(E) emit : absorb
The best answer is C.
Similar pairs: instruct : goad :: tend : fuss :: reproach : upbraid :: like : dote :: suggest : urge :: drink :
guzzle :: embarrass : mortify :: glimmer : dazzle :: warm : sear :: involve : entangle :: trickle : gush ::
gnaw : nibble :: quaff : sip :: gobble : nibble :: suffuse : tint :: wander : amble :: enrage : irk.
Noun to noun
TWIG : LIMB ::
(A) ally : felon
(B) invention : discovery
(C) adversary : mediator
(D) pebble : boulder
(E) dispute : disagreement
66

The best answer is D.


Similar pairs: quibble : objection :: foible : flaw :: figurine : colossus :: storm : hurricane :: fire :
inferno :: tiff : quarrel :: minutiae : details :: elapse : error :: nuance : distinction :: inkling :
indication :: admonishment : castigation :: irreverence : blasphemy :: confusion : delirium ::
unconsciousness : coma :: peccadillo : sin :: ecstasy : pleasure :: surprise : astonishment :: and
pride : hubris.
Fast to slow
LOPE : RUN ::
(A) broach : track
(B) premeditate : precipitate
(C) promote : educate
(D) prevent : broke
(E) drawl : speak
The best answer is E.
Similar pairs: grow : burgeon :: beat : palpitate :: crawl : proceed :: dwindle : decrease.
Normal to unnormal
BABBLE : TALK ::
(A) scribble : write
(B) snow : rain
(C) smooth : broach
(D) attract : impress
(E) sum : calculate
The best answer is A.
Similar pairs: simper : smile :: lurk : wait :: abscond : depart :: secret : store :: pirate : take :: slur :
speech :: smudge : writing :: prate : speak :: saunter : walk :: infiltrate : enter and insinuate : say.
Sudden to general
CONTRACT : IMPLODE ::
(A) generate : born
(B) fall : plummet
(C) assuage : relieve
(D) depend : socialize
(E) deprive : restate
The best answer is B.
Similar pairs: turn : swerve.
Force to general

67

SUPPLANT : REPLACE ::
(A) frame : plan
(B) incinerate : corrode
(C) snatch : take
(D) evaporate : petrify
(E) sing : dance
The best answer is C.
Similar pairs: importune : request :: pry : inquire :: coercion : persuade :: exile : emigrate ::
command : request :: conscript : enlist.
4. Person to character
Person to personality (Positive)
ZEALOT : FERVOR ::
(A) pundit : stickiness
(B) actor : popularity
(C) singer : beauty
(D) altruist : selflessness
(E) editor : comment
The best answer is D.
Similar pairs: partisan : allegiance :: diplomat : tact :: inventor : ingenuity :: coward : craven :: dupe :
credulous :: acrobat : agility :: boor : insensitive :: loner : solitary :: surgeon : dexterity :: blowhard :
boastful :: toady : obsequious :: supplicant : humility :: adversary : resistance :: recluse :
withdrawn :: bigot : biased :: wag : humorous :: dolt : stupid.
Person to personality (Negative)
MAVERICK : CONFORMITY ::
(A) expert : authoritativeness
(B) acrobat : agility
(C) evaluator : critiques
(D) statistician : accuracy
(E) stickler : approximation
The best answer is E.
Similar pairs: purist : adulteration :: heretic : orthodoxy :: poseur : sincerity :: recluse :
gregariousness :: coward : brave :: philanthropist : selfish :: neophyte : experience :: boor :
sensitivity :: yokel : sophistication.
Person to behavior
PROCTOR : SUPERVISE ::

68

(A) prodigal : squander


(B) director : act
(C) speaker : present
(D) trader : market
(E) driver : deliver
The best answer is A.
Similar pairs: conspirators : collusion :: juggernaut : crush :: quisling : betray :: foragers : grazing ::
hunter : stalking :: faultfinder : criticize :: arbitrator : mediate :: instigator : incite :: bully : browbeat.
Person to aspiration
HEDONIST : PLEASURE ::
(A) manager : leadership
(B) recluse : privacy
(C) merchant : commodity
(D) educator : publication
(E) historian : document
The best answer is B.
Similar pairs: ascetic : self-control.
Person to character
PUNDITS : AUTHORITATIVENESS ::
(A) instructor : presentation
(B) archeologist : history
(C) expert : expertise
(D) inventor : imagination
(E) writer : publication
The best answer is C.
Similar pairs: pest : irksome.
5. Part to whole
Here, the two words are similar in category, but one is the part of the other.
STANZA : POEM ::
(A) student : school
(B) snow : precipitation
(C) fresco : wall
(D) courtyard : fountain
(E) act : opera

69

The best answer is E.


Sometimes, the relationship of part to whole can be defined more narrowly. For example, credit to
movie is as byline to article. Here, a credit appears in the beginning of a movie jus as a byline
appears in the beginning of an article. Other pairs of words that have similar relationship include:
preamble : statute :: overture : opera :: prologue : play :: prelude : music :: coda : sonata. Also, the
following pairs of words have the relationship of part to whole:
leaf : oak :: needle : pine :: frond : fern.
6. Individual to all
This type of relationship is similar to the part to whole, but it is different in that in this relationship,
one word is an independent element of the category described by the other word while in the
relationship of part to whole, one word is a dependent part of the other.
CHOIR : SINGER ::
(A) cast : actor
(B) team : leader
(C) employee : operator
(D) human : animal
(E) professor : expert
The best answer is A.
Similar pairs: orchestra : instrumentalist :: flock : birds :: school : fish :: herd : cattle :: soldier :
army :: colony : bacterium :: armada : vehicle :: fusillade : projectiles :: barrage : explosives ::
gravel : pebble :: nation : citizen.
Sometimes, you should consider the way in which individuals are formed into category. For
example, matrix to number is as crystal to atom, but is not as gas to molecule since there is some
spatial order in forming number to matrix, and there is not for gas to molecule.
When the individual is plural in number, the relationship turns into synonym. For example, people :
persons :: staff : workers :: faculty : teachers :: portfolio : securities :: dossier : report and clientele :
customers.
7. Member to class
In this type of relationship, one word is a kind of the other.
LIMOUSINE : AUTOMOBILE ::
(A) door : window
(B) mansion : residence
70

(C) act : opera


(D) school : fish
(E) crystal : atom
A rationale for this analogy could be X is a large, expensive member of class Y. The best
answer is B.
Similar pairs: turquoise : gem :: snow : precipitation :: violet : flower :: compendium : summary ::
anthology : collection :: cacophony : sound :: lampoon : satire :: sonnet : poem :: royalty :
payment :: tango : dance :: bowl : receptacle :: glare : light :: granite : rock :: magazine : periodical ::
gust : wind :: cloudburst : rainfall :: mimicry : camouflage :: mutation : variation :: enzyme :
catalyst :: bacterium : microbe :: ostrich : bird :: tiger : cat :: suitcase : luggage :: hat : millinery.
8. Actor to action
One word names an actor, and the other describes his/her action.
CURATOR : ART ::
(A) historian : story
(B) conductor : singer
(C) archivist : document
(D) president : citizen
(E) lawyer : right
The best answer is C.
Similar pairs: conductor to symphony is as director to film.
9. Actor to object
One word names an actor, and the other describes his/her object.
MISANTHROPE : PEOPLE ::
(A) trader : company
(B) astronomer: sky
(C) painter : art
(D) xenophobe : stranger
(E) employer : employee
The best answer is D.
Similar pairs: painter : canvas :: sculptor : marble :: miller : grain :: tanner : hide :: referee : whistle ::
judge : gavel :: impresario : entertainment :: broker : trade :: astronomer : star :: spelunker : cavern.
10. Actor to tools
CARPENTER : SAW ::
71

(A) judge : law


(B) author : book
(C) actor : movie
(D) artist : picture
(E) surgeon : scalpel
The best answer is E.
Similar pairs: tailor : scissors :: bricklayer : trowel :: astronomer : telescope :: butcher : knife.
11. Action to sense
OBEISANCE : SUBMISSION ::
(A) countenance : toleration
(B) regression : termination
(C) unconventionality : tradition
(D) abundance : corporate
(E) arm : force
The best answer is A.
Similar pairs: sneer : contemptuous :: gush : effusiveness :: rage : irate :: blush : discomfited ::
gloat : smug :: groan : aggrieved :: cower : fear :: swagger : bravado :: snub : disdain :: demur :
objection/qualms :: embrace : affection :: frown : displeasure :: preen : self-satisfaction :: fume :
anger :: apologize : contrite :: compliment : impressed :: exult : satisfaction :: crave : longing.
12. Action to object
ACCELERATE : SPEED ::
(A) fly : height
(B) prolong : duration
(C) run : distance
(D) accomplish : assignment
(E) present : comment
The best answer is B.
Similar pairs: nurture : child is as cultivate : crop :: assuage : sorrow is as damper : ardor :: carve :
turkey is as slice : cake :: parry : question is as shirk : duty :: barter : commodities is as correspond :
letters :: prune : hedge is as trim : hair :: damp : vibration is as stanch : flow :: disbar : attorney is as
expel : student :: dally : time is as squander : money :: decipher : hieroglyph is as break : code ::
splice : rope is as weld : metal :: molt : feather is as shed : hair :: countermand : order is as revoke :
license :: reprieve : punishment is as moratorium : activity :: embezzle : fund is as usurp : power.
13. Cause to effect

72

As its name suggests, one word causes the other. The relationship between these two words is
cause and effect.
TORQUE : ROTATION ::
(A) rain : wetness
(B) snow : precipitation
(C) tension : elongation
(D) school : education
(E) eulogy : praise
The best answer is C.
Similar pairs: redoubtable : awe :: venerable : respect :: despicable : contempt :: droll : laugh ::
pernicious : injure :: disingenuous : mislead :: dreadful : cringe :: nervousness : fidget :: macabre :
shudder :: hilarious : laugh :: motive : deed :: doubt : question.
For example: evaporate : vapor :: freeze : ice :: incinerate : ash :: petrify : stone :: sequester :
seclusion :: endanger : jeopardy :: ripen : maturity :: harden : solidity.
Also: homogenization : uniform :: putrefaction : rotten :: corrosion : rust :: condensation : dew ::
combustion : soot :: woodcutting : sawdust.
More example: numb : insensible :: burnish : lustrous.
For example: illuminate : darkness :: educate : ignorance :: embellish : austere :: adulterate : pure ::
renounce : pledge :: retract : statement :: repeal : law :: rescind : order :: withdraw : candidacy ::
perturb : serenity :: reassure : doubt :: neutralization : acid :: disabuse : error :: rehabilitate :
addiction :: detoxify : poison :: dehydrate : water :: purify : imperfection :: verify : doubtfulness ::
expiate : guilt :: correct : error :: slake : thirsty :: satiate : hunger.
14. Modification
In this type of relationship, an adjective modifies a noun. A similar pair of words should have the
relationship in the same order, and in same tone (positive or negative).
LACONIC : SPEECH ::
(A) mysterious : demonstration
(B) detailed : generation
(C) popular : foundation
(D) austere : design
(E) many: atom
The best answer is D.
Similar pairs: articulate : speech is as graceful : movement :: volatile : temper is as ready : wit ::
frenetic : movement is as fanatical : belief :: fetid : smell is as ugly : appearance :: stridency : sound
is as garishness : appearance ::

scoop : concave is as spatula : flat or skewer : tined.


73

15. Symbol to representation


In this type of analogy, one word represents the other.
CORNUCOPIA : ABUNDANCE ::
(A) pebble : stone
(B) snow : precipitation
(C) actor : movie
(D) school : fish
(E) mace : authority
The best answer is E.
Similar pairs: trademark : company :: ensign : country :: sacrifice : worship :: augury : prediction.
Other pairs: parenthesis : explanation :: ellipsis : omission :: asterisk : annotation :: caret : insert ::
comma : pauseseparate :: bracket : enclose :: hyphen : join :: period : over :: colon : emphasis
attention :: underscore : emphasis.
16. Function
In this type of analogy, one word serves as the function of the other.
PANEGYRIC : EULOGIZE ::
(A) lampoon : satirize
(B) style : diversify
(C) elegy : admiration
(D) analgesic : ridicule
(E) sextant : captain
The best answer is A.
Similar pairs: tirade : criticize :: hymn : praise :: dirge : grief :: eulogy : admiration :: elegy :
sorrow/lament :: autobiography : reminisce :: anecdote : amusement :: lecture : instruction.
For example: ballast : stabilize :: astringent : pucker :: spark : ignition :: bacteria : decomposition ::
yeast : fermentation :: stockade : enclosure :: sedative : pacify :: antiseptic : sterilize :: emollient :
soothe :: dynamo : generate :: epithet : disparage :: alias : mislead :: desiccant : dry :: caustic : eat
away :: food : nourish :: antibodies : protect :: frieze : ornament :: pillar : support :: sponge :
absorb :: guillotine : execute :: analgesic : deaden :: pillory : ridicule.
For example: glue : attaching is as ink : printing :: drill : boring :: die : shaping :: needle : knit ::
loom : weave :: abacus : calculate :: sextant : navigate :: yarn : weave.
17. Thing to character

74

WHIM : CAPRICIOUS ::
(A) story : runic
(B) fact : objective
(C) policeman : active
(D) runner : strong
(E) crystal : beautiful
The best answer is B.
Similar pairs: synopsis : condensed :: plant : herbaceous :: trees : arboreal :: orchestra :
instrumental :: labyrinth : tortuous :: ornament : decorative :: gadfly : annoying :: stealth : furtive ::
caprice : whimsical :: bombast : pompous :: tirade : critical.
Negative: chance : inevitable :: invention : insipid :: imagination : prosaic :: gaucherie : urbane ::
melodrama : subtlety :: chimera : authenticity.
18. Material to product
FILIGREE : WIRE ::
(A) antibiotic : drug
(B) coagulant : clearness
(C) lace : thread
(D) concert : song
(E) atom : crystal
The best answer is C.
Similar pairs: candle : wax :: wood : paper :: porcelain : clay :: or tornado : air :: whirlpool : water.
19. Other relationship
1). Thing to skin. For example: apple to skin, melon to rind, conifer to back, pecan to shell, wheat
to chaff, mammal to epidermis, seed to hull, cell to membrane.
2). Thing to scrap. For example: crumb to bread, shard to pottery, shaving/splinter to wood, filing
to metal, fragment to bone.
3). Block. For example: tourniquet to blood, dam to water.
4). Place. For example: envelope to letter, crate to produce, larder to food, armory to munitions,
hamper to laundry, bookcase to books, wardrobe to clothes.
5). Elimination. For example: antidote to poison, tonic to lethargy.
6). Support. For example: strut to wing, buttress to wall, bone to body, guy to pylon, framing to
building, girder to skyscraper, skeleton to animal.

75

Chapter 4 Antonyms
There are about 9 antonyms on verbal section of the GRE. The questions are mixed in with the
analogies, sentence completions, and reading comprehension. In antonym question, you are
given a capitalized word, and then required a pick up a word among five that is opposite or nearly
opposite to the original word in meaning. A sample antonym questions looks like this:
PROFOUND
(A) superficial
(B) precipitous
(C) deep
(D) tarnished
(E) innocuous
Which of the words is opposite in meaning to the word profound? The word profound can be
adjective or noun. Should we consider it in two functions? The answer is that the five words
should be considered in same function. Here it is an adjective. Profound means difficult to
understand or far below the surface. In choice A, superficial means lying on surface, directly
opposite to the meaning of profound. Precipitous means steep in rise or fall, deep means
same as profound, tarnished means losing luster, and innocuous means harmless. Among
the five words, only superficial has the meanings that are opposite to the given capitalized.
Trap: Be alert to secondary meaning of a word. The GRE writers often use common words but
with its uncommon meaning.

Put the words in sentence

A single word is difficult to understand, when combined into phrases or sentences, however, we
have little trouble. If you dont recognize the meaning of a word, think of a phrase in which you
have hear it before.

Consider the root word

You may not know a given word, but you can spot the root word to deduce the meaning of the
original word. Most words are derived from other words.

76

Vocabulary Roots
By doing some brainstorming, you can substantially boost your vocabulary base. If you have
limited time to prep for GRE or have few weeks before the test, you can try to memorize a list of
roosts. On the test day, you can guess by those roots.

ROOT

MEANING

EXAMPLE

act, ag, ig

act, drive

interact, counteract, agitate, activate

anim

breath, mind

cad, cas, cid

fall

cascade, coincidence, casualty

cap, cep

receive, take

capacious, captivate, perceptible

cede, cess

go, yield

antecede, cession, intercede, precede

cid, cis

cut, kill

concise, excise, scission, suicide

cit

excite, call

recital, solicit, solicitous

claim, clam

cry out

acclaim, clamorous, proclaim, disclaim

clud, clus

close

conclude, occlude, reclusive, secluded

10

cor, cord, cour

hart, courage

concordant, discourage, discord

11

corp, corpor

body

corporeal, corpulent, incorporate

12

cours, cur, curs

run

13

cred

believe

14

dic, dict

say

15

duc, duct

lead

animate, animosity, equanimity,


pusillanimous

concourse, precursor, excursion,


incursion
accredit, creditable, miscreant, credulous
abdicate, benediction, malediction,
jurisdiction
abduct, aqueduct, seduce, subdue

77

16

17

equ

fac, fact, fair, feat,


fect, fic, fict

equal

do, make

adequate, equilibrium, equivocal,


equipoise
affair, affectation, artifact, beneficiary,
factor, infectious, magnificent

18

fer

carry, bear

circumference, fertile, transfer

19

flu

flow

affluence, influential, superfluous

20

grad, gress

go, step

aggressive, ingredient, retrogressive

21

join, joint, junct

join, connect

adjunct, conjuncture, subjoin

22

locu, log, loqu

speak

allocution, circumlocution, grandiloquent

23

luc, lumin, lustr

light, brightness

elucidate, luminous, translucent

24

magn, maj, max

great

magnanimous, majesty, magnitude

25

main, man

hand

emancipate, manacle, manuscript

26

mini, minu

small

miniature, diminutive, diminish

27

mob, mom, mot, mov

move

commotion, momentous, immobile

28

pass, path

feel, suffer

antipathetic, apathetic, sympathetic

29

ped

foot

pedestrian, impediment, expeditious

30

pel, puls

drive, push

compulsory, impulsive, repulsive

31

vers, vert

turn

divert, inverse, controversy

32

vig, vit, viv

life

invigorate, vitalize, vivid

33

voc, vok

voice, call

advocate, irrevocable, vociferate

34

volu, volv

roll

intervolve, voluminous, revolt

78

PREFIX

MEANING

EXAMPLE

ab, abs

away, from

abdicate, abjure, abscond, abominate

ad

to, toward

adulterate, adhere, adjourn

bene

good

beneficial, benevolent, benignant

cata

down, complete

cataract, cataclysm, catastrophe

circum

around

co, com, con

with, together

contra, contro,
counter

against

circumference, circumlocution,
circumstantial
coalesce, coherent, coincide,
compassion, consensus
contradict, counterattack, countermand,
controversy, counterpart, counterfeit

de

down, complete

debase, depress, deduct

de, di, dis

away, off, not

decipher, desperate, detain, decode

10

dia

through, between,

diacritic, diagonal, diaphanous, diatribe,

across

diagram

11

e, ec, ex, extra

out

12

em, en, im, in

in, upon

embellish, immanent, environment

13

fore

before

aforementioned, foredoom, forebear

14

inter

between, among

15

mal

bad

malcontent, malevolent, malnutrition

16

mon, mono

one

monarch, monotonous, monolog

17

para

beside, beyond

paradox, paraphrase, paragon

ebullient, eccentric, effectuate, emissary,


exacerbate, exaggerate, explicable

interaction, interrogate, intermission,


intermingle

79

through,

18

per

19

post

after, behind

postpone, postscript, posterity

20

pre

before

precede, prelude, presage

21

re

back, again

rebound, rehabilitate, rehearse

22

se

away, separate

secede, segregate, sever

23

sub, sus

under

subdivide, suffocate, subsequent

24

super, sur

over, above

superabundant, surfeit, surplus

25

sym, syn

with, together

sympathy, synthesis, syndicate

26

trans

27

uni

one

unanimous, unison, uniform

SUFFIX

MEANING

EXAMPLE

ain

people

villain, chaplain

aire

people

millionaire, billionaire

an, ian, ese

people

German, barbarian, Chinese

-ary

people

secretary, adversary

ee

people

employee, nominee , referee

eer

people

engineer, pioneer, volunteer

ess

people

actress, empress, hostess

-ician

people

electrician, musician, physician

ist

people

typist, antagonist, dogmatist

thoroughly

across, through,
over

percolate, perspicuous, pertinacious

transcend, translucent, transitory

80

10

cle

small

miracle, particle, speckle

11

cule

small

minuscule, molecule, granules

12

el

small

model, pebble

13

en

small

kitten, maiden

14

et

small

cabinet, islet

15

let

small

leaflet, pamphlet, hamlet

16

proof

anti

waterproof, weatherproof, foolproof

17

ward

direction

backward, awkward, wayward

The End of GRE Verbal Study Guide

81

Вам также может понравиться